CO2andTempKalteSonne

Bild 1: Entnommen aus der “Kalten Sonne” von der RWE Klimagruppe  Vahrenholt/Lüning zeigt diese Kurve Daten aus Monin et al. 2001 mit einem Verzug von 800 Jahren zwischen CO2 und antarktischer Temperatur.

 

3)   Dann, vor mittlerweile auch schon wieder einem Jahr, gab es ein Paper von Jeremy Shakun der Temperatur-Zeitserien basierend auf den verschiedensten Proxys und verteilt über den ganzen Planeten mit den Eis- und Gas-Zeitserien aus der Antarktis verglich. In der Schlüsselgrafik (siehe Bild 2) erkennt man, dass die Südhemisphäre dem CO2 Anstieg vorläuft, es also dort zuerst wärmer zum Ende der letzten Eiszeit wurde (Termination I), dann das CO2 anstieg und dann auch der Rest des Planeten (Tropen und Nordhemisphäre) erwärmte. Mein Problem damit, war eigentlich nur auf der Seite des Kohlenstoffzyklus, nicht auf der Seite der Klimadynamik. Das verspätet austretende CO2 hat eben mit einem Verzug von ein paar 100 Jahren in das Spiel der Feedbacks eingegriffen und hauptsächlich dazu beigetragen, die Regionen ausserhalb der mittleren und hohen Breiten der südlichen Hemisphäre zu erwärmen. Kein Problem. Was mich aber störte oder zumindest etwas überraschte ist die lange Zeit, in der der sich erwärmende südliche Ozean angefangen haben soll, das CO2 auszugasen. 600-800 Jahre sind doch eine lange Zeit. Der Aufschrei in der klimaskeptischen Gemeinde war einhellig (hier etwa der RWE Klimakaspar) und die Argumente zu Shakun auf dem gewohnten Niveau. Wie konnte es sein, dass einer ihnen dieses geliebte Argument wegnehmen konnte, dass das CO2 der Temperatur folge und nicht umgekehrt? Unerträglich! Selbst wenn das unterschiedliche Timing all dieser Zeitserien zum Ende der letzten Eiszeit längst vorher bekannt war.

 

4)   Und jetzt kommt es noch schlimmer. Und zwar so schlimm anscheinend, dass absolut keiner der üblichen Verdächtigen auch nur das geringste Wort dazu verliert. Basierend auf neuen und ziemlich umfangreichen Eismessungen eines anderen Gases, nämlich Stickstoff, und seiner Isotope ist es der gleichen Gruppe von meinem ehemaligen Labor, dem LSCE,  die auch schon das Caillon et al. Paper herausgebracht haben, gelungen, zu einer genaueren Aussage bzgl. des Timings von Temperatur und CO2 zu kommen. Das 15N gibt eine fast kontinuierliche Abschätzung der sogenannten Abschlusstiefe (also die Tiefe des Firn, in der sich die Poren schliessen und das Gas nicht mehr mit der Atmosphäre austauschen kann).

Diese Tiefenabschätzung wurde mit anderen unabhängigen Methoden geprüft und das schliessliche Resultat sieht man in Bild 3. Es gibt keinen signifikanten Verzug mehr zwischen CO2 und den Temperaturen der Antarktis  und auch die Methanmessungen in der Nord- und in der Südhemisphäre sind nun in perfekter Synchronisation. Die globale Enteisung der letzten Eiszeit begann in der Antarktis ab etwa  18.000 Jahren vor heute und war praktisch instantan von entsprechenden CO2 Änderungen begleitet. Gleiches gilt für alle folgenden Klimaschwankungen während des Übergangs zur holozänen Warmzeit.

 

 

 

NorthSouthHemisphRec

Bild 2: Grafik aus Shakun et al. 2012. Es zeigt das CO2 und verschieden global verteilte Temperaturproxies. In dieser Grafik wurde noch angenommen, dass das CO2 im antarktischen Eis dem CO2 ca 800 vorläuft. Die gestrichelte Lag 0 Linie im unteren Bild, welches Temperaturproxies aus Süd- und Nordhemisphäre relativ zum CO2 zeigt, müsste nach den neusten Ergebnissen der Eiskerngruppe aus Frankreich also 800 Jahre nach links verschoben werden.

 

Was sind die Konsequenzen dieses neuen Papers?

 

1)   Das Monnin et al Paper  (siehe Bild 1) hatte noch einen Verzug des CO2 von ca. 800 Jahren für die gleiche Temination I berechnet. Es basierte auf einer Modellrechnung, die es erlaubte die Porenabschlusstiefe direkt aus Temperatur und Akkumulation zu berechnen. Es sieht so aus, dass diesem Firnmodell Elemente fehlen. Im Papier hier wird über den Grad zusätzlicher Verschmutzung des Eises durch Staub spekuliert, welcher einen Einfluss auf Porenbildung, Größe, etc. nehmen könnte. Mehr oder weniger Staub hätte dann einen unterschiedlichen Einfluss auf die insgesamt 8 Eiszeit/Zwischeneiszeit Übergänge, die mittlerweile in den Eiskernen dokumentiert sind. Das wird noch spannend.

1 / 2 / 3

Kommentare (158)

  1. #1 Chemiker
    März 8, 2013

    ist (hier und hier und in verschiedenen Skeptikerblogs)

    Der Logik nach sollte „hier“ wohl ein Link sein, ist es aber nicht.

  2. #2 Georg Hoffmann
    März 8, 2013

    Mein Gott, seid ihr schnell. Ich bin noch beim Korrigieren. Links sind schon gesetzt. Danke.

  3. #3 Chemiker
    März 8, 2013

    RSS 😉

  4. #4 axel
    März 8, 2013

    Ja, Shakun. In der Skeptikerszene folgte auf Shakuns Paper erst ein Aufschrei der Empörung (alleine 6-7 Beiträge in 14 Tagen nur bei WUWT, von “Widerlegungen” bis Verdächtigungen gegen Shakun alles dabei). Und dann?

    Schweigen. Als hätte es niemals Shakun gegeben. CO2 folgt immer der Temperatur, heißt es bis heute. Verdrängung zur Erhaltung des eigenen Glaubensdogmas, lautet meine laienpsychologische Ferndiagnose.

    Vielleicht kann ja Krishna mal exemplarisch seine Probleme mit der Anpassung an neue Daten erklären…

    PS:
    Bild 3 = Bild X? Wo ist es? 😉

  5. #5 Georg Hoffmann
    März 8, 2013

    Sorry fuer die vielen Fehler. Ich wollte den Beitrag noch weiter korrigieren und dann kamen die Kinder und dann dies und dann das.. Wie’s halt so geht. Jetzt sollte es etwas besser sein.

  6. #6 axel
    März 8, 2013

    Watts hat gerade ein anderes Problem.
    Und wieder ist Shakun mit dabei…

  7. #7 Dr. Webbaer
    März 9, 2013

    @Hoffmann
    Ein paar Fragen, vielleicht haben Sie Lust & Zeit zu antworten, Frage Nr.1:
    Frage zum Carbonkreislauf – wenn es wärmer wird, reagiert die Biosphäre u.a. dadurch, dass die Tierwelt weniger Carbon freisetzt als die Pflanzenwelt mehr aufnimmt, so dass langfristig (all diese Betrachtungen sind langfristiger Art) der atmosphärische CO2-Gehalt wieder sinkt?

    MFG
    Dr. W (der nicht ohne Grund eine kleine Fragenserie vorbereitet hat)

  8. #8 Georg Hoffmann
    März 9, 2013

    @WB
    Na, dann mal los. Dafuer ist der Blog ja da.
    Tiere sind nur ein winziger, irrelevanter ANteil des Kohlenstoffzyklus. Ausserdem haben Tiere einen sehr schnellen Turnover. Ist die Maus erstmal tot, ist sie auch schon ruckzuck zersetzt. Im Gegensatz zu Torfen, Mooren oder allgemein Cellulose haltiger organischer Masse. Man kann also Tier).e vernachlaessigen.
    Im uebrigen (ich bin nicht sicher, ob Sie das meinten) ist die Kohlenstoffbilanz eines Tieres fast null (zumindest, wenn es erstmal ausgewachsen ist.

  9. #9 Dr. Webbaer
    März 9, 2013

    @Hoffmann
    Danke, Frage Nr. 2:
    Wenn die Biophäre anzunehmenderweise nicht für die bekannten Schwankungen beim atmosphärischen CO2 und damit nicht anzunehmernderweise für die bisher beobachteten oder belegten Klimaschwankungen verantwortlich ist [1], weil wir bspw. bestimmte Mutationen auf bakterieller Ebene oder tierische Besonderheiten wie die gedankenexperimentell erstellte Furzkuh [2] ausschließen: Welche Besonderheiten waren dann bisher verantwortlich für die festgestellten Temperaturschwankungen die terrestrischen Oberflächentemperaturen betreffend?

    MFG
    Dr. W

    [1] gemeint der Carbonkreislauf oder andere Kreisläufe klimarelevante Gase betreffend
    [2] die Furzkuh unterscheidet sich nicht grundsätzlich vom jetzt u.a. carbonlösenden Primaten

  10. #10 Thomas
    März 9, 2013

    Selbst wenn es wahr wäre, dass CO2 der Temperatur 800 Jahre hinterherhinkt, wäre dies ja ein verdammt gutes Argument, den Ausstoss von CO2 möglichst schnell herunterzufahren, da uns sonst unsere Nachkommen in 800 Jahren verfluchen werden, da wir es ja gewusst und nichts getan haben, denn ihnen wird all das Zeugs um die Ohren fliegen. Jetzt sind es halt nur 200 Jahre.

    Aber das ist halt so mit den Klima “Skeptikern”. Einerseits ist die Sensitivität so gering, das eine Verdoppelung der CO2 Konzentration kein Problem sein wird, andererseits ist sie so hoch, dass die viel kleineren Forcings der letzten 1000 Jahre sowohl die “Mittelalterliche Warmzeit” als auch die “Kleine Eiszeit” hervorgebracht haben

  11. #11 Günther Vennecke
    März 9, 2013

    @Baerchen (der offensichtlich mal wieder nichts verstanden hat)

    “wenn es wärmer wird, reagiert die Biosphäre u.a. dadurch, dass die Tierwelt weniger Carbon freisetzt als die Pflanzenwelt mehr aufnimmt…?”

    Das ist dann doch etwas sehr verkürzt ausgedrückt. Erst einmal dürfte die Tierwelt zur globalen CO2-Konzentration nur einen verschwindend geringen Beitrag leisten (einfach mangels Masse), zweitens ist die Biomasseprodution der Pflanzen von sehr viel mehr Faktoren abhängig als nur von der CO2-Konzentration und drittens spielen die Weltmeere die mit Abstandgrößte Rolle, wenn es darum geht, wie viel CO2 in der Atmosphäre verbleibt.

    Und da wird es dann wieder relativ einfach (aber für Baerchen immer noch drei Nummern zu hoch): das Gleichgewicht (sofern man in einem dynamischen System wie Atomsphäre Hydrosphäre überhaupt von Gleichgewicht sprechen kann) ist hauptsächlich temperaturbestimmt und zwar dergestalt, dass mit zunehmender Temperatur die Aufnahme von CO2 ins Meerwasser verlangsamt wird. (Möglicherweise wissen sogar Sie, dass die Meere eine CO2-Senke darstellen). Dadurch kann bei entsprechenden CO2-Quellen der CO2-Gehalt der Atmosphäre dann ansteigen.

    Bei abnehmender Temperatur verhält es sich dann entsprechend umgekehrt.

    Diese grundlegenden Vorgänge können natürlich durch andere Prozesse überlagert werden, so dass sich insgesamt ein relativ komplexes Bild ergibt, so komplex, dass damit praktisch alle Klimaleugner völlig überfordert sind..

  12. #12 Dr. Webbaer
    März 9, 2013

    @Vennecke
    Dr. W weiß das alles, betrachten Sie die Fragestellungen bitte nicht als Meinungsäußerungen.

  13. #13 Günther Vennecke
    März 9, 2013

    @Baerchen,

    Sie versuchen hier zu trollen. Solche Fragen finden Sie im Web auf Tausenden von Sites beantwortet.

    Oder Sie kaufen sich einfach ein ordentliches Buch, das mit wissenschaftlichem Anspruch das Thema Klima behandelt.

    Erlaubt sein muss aber die Frage: Wenn Sie sooooooooooo wenig Ahnung von der Sache an sich haben, warum wollen Sie sich denn überhaupt an der Diskussion beteiligen? Außer ein paar dämlichen Fragen können Sie doch eh nichts beitragen.

    Sie haben doch schon in der Vergangenheit zur Genüge gezeigt, dass es bei Ihnen nicht allzu weit reicht.

  14. #14 Dr. Webbaer
    März 9, 2013

    @Vennecke

    Solche Fragen finden Sie im Web auf Tausenden von Sites beantwortet.

    Nicht in der beim Anfragenden vorliegenden Serie.

    Gedulden Sie sich bitte noch ein wenig.

    MFG
    Dr. W

  15. #15 Günther Vennecke
    März 9, 2013

    Meine Liebingsstelle in dem Propaganda-Lügen-Film “The great global warming swindle” ist immer noch der Auftritt des Hochstaplers und notorischen Lügners Tim Ball, der auch hier wieder seine überaus hohe “Qualifikation” dadurch belegt, dass er nicht einmal die CO2-Daten richtig benennen kann.

    Tja, liebe Klimaleugner, wer solche Freunde hat, braucht nun wirklich keine Feinde mehr. 🙂 🙂 🙂

    Das ist ja überhaupt das Schöne am Internet: man braucht nur ein wenig herum zu surfen und zu schauen, was die einzelnen Leute so von sich geben (vorzugsweise Originaldokumente und Artikel aus seriösen Zeitungen) und mit wem sie sonst so Kontakt haben, dann wird einem sehr schnell klar, was Sache ist.

    So entsteht sehr schnell eine Wirk-Kette, die ein eindeutiges Ergebnis liefert, z. B.,

    Ball tritt mit Singer auf, der mit Monckton gemeinsame Auftritte hat. Monckton und SInger sind mit eike, dem sattsam bekannten Verein verlogener Tattergreise liiert. eike und cfact sind eng miteinander verwoben, letztere entstammen einer Allianz aus Ultrakonservativen Republikanern in den USA und der dortigen Energie-Lobby, die wiederum Leuten wie Singer, Carter, Plimer, usw. höhere Geldbeträge zukommen lässt. Noch Fragen?

  16. #16 Günther Vennecke
    März 9, 2013

    Ach Baerchen,

    “Nicht in der beim Anfragenden vorliegenden Serie.”

    Bisher war jedenfalls nichts originelles dabei, nur Fragen, die verraten, dass der Fragen Stellende extrem wenig von der Materie versteht.

  17. #17 Rahmhuber
    März 9, 2013

    “…Diese grundlegenden Vorgänge können natürlich durch andere Prozesse überlagert werden, so dass sich insgesamt ein relativ komplexes Bild ergibt, so komplex, dass damit praktisch jeder behaupten kann, was er will und ihm in den Kram passt..”
    Das ist, glaub ich alles, was schlussendlich beim “Normalbuerger” noch ankommt, und niemanden mehr interessiert.

  18. #18 Georg Hoffmann
    März 9, 2013

    @wb
    “Wenn die Biophäre anzunehmenderweise nicht für die bekannten Schwankungen beim atmosphärischen CO2 ”

    Da haben Sie meine Antwort falsch verstanden. Ich habe zur Zoosphaere gesprochen. Die Biosphaere im allgemeinen hat mit Sicherheit einen Beitrag zur CO2 Aenderung beim Wechsel von Eiszeit zu Zwischeneiszeit gehabt. Verschiedene Abschaetzungen gehen davon aus dass ~1000-1500 GTC (GigaTonnen C) nach Ende der letzten Eiszeit in die nachwachsenden Waelder eingegangen sind. Das macht also das Problem noch groesser. Man muss nicht nur die 100ppm Anstieg (die sie oben auch in Bild 3 sehen) erklaeren sondern obendrein die 1000 GTC die in die Biosphaere geflossen sind.

    “Welche Besonderheiten waren dann bisher verantwortlich für die festgestellten Temperaturschwankungen die terrestrischen Oberflächentemperaturen betreffend?”

    Erdbahnschwankungen, CO2, CH4, Vegetation mit seiner Albedo, Staub sind so die wichtigsten.

  19. #19 Georg Hoffmann
    März 9, 2013

    @Thomas
    “Aber das ist halt so mit den Klima “Skeptikern”. Einerseits ist die Sensitivität so gering, das eine Verdoppelung der CO2 Konzentration kein Problem sein wird, andererseits ist sie so hoch, dass die viel kleineren Forcings der letzten 1000 Jahre sowohl die “Mittelalterliche Warmzeit” als auch die “Kleine Eiszeit” hervorgebracht haben”
    Ha! Gut erkannt.

  20. #20 Günther Vennecke
    März 9, 2013

    @Rahmhuber,

    ““…Diese grundlegenden Vorgänge können natürlich durch andere Prozesse überlagert werden, so dass sich insgesamt ein relativ komplexes Bild ergibt, so komplex, dass damit praktisch jeder behaupten kann, was er will und ihm in den Kram passt.””
    (Der hervorgehobene Teil des Zitats wurde von Rahmhuber gefälscht.)

    das Tatsache, dass Sie mein Zitat in obiger Weise verfälschen, belegt genau die Überforderung des Klimaleugners Rahmhuber.

    Wieder einmal zeigen Sie Ihre tief sitzende Ablehnung der Wissenschaft, weil sie Ergebnisse liefert, die Ihnen nicht in den Kram passen. Aber trösten Sie sich, diese Eigenschaft teilen Sie ja mit allen anderen Klimaleugnern. Besonders schön kommt diese Ablehnung der Wissenschaft ja im Namen der Website “science-skeptical” zum Ausdruck. 🙂

  21. #21 Dr. Webbaer
    März 9, 2013

    @Hoffmann
    Sie waren in Frage 1 schon zur Biosphäre befragt worden, Ihre Antwort bezog sich -Ihren Angaben entsprechend- also auf die Zoosphäre, der Schreiber dieser Zeilen fragt denn einfach mal, sozusagen rückstufend als Frage 1b nach dem Einfluss der Biosphäre noch einmal: Eher gering, “Eiszeiten” einleitend oder wie würden Sie dementsprechend und reduzierend einordnen wollen?

    Frage 2 darf aber auch gerne schon einmal, sozusagen abschliessend, beantwortet werden. Falls möglich.

    MFG
    Dr. W (der sich frühestens am 11.03.2013 wieder melden wird)

  22. #22 Georg Hoffmann
    März 9, 2013

    @WB
    Als noch einmal
    “Frage Nr.1:
    Frage zum Carbonkreislauf – wenn es wärmer wird, reagiert die Biosphäre u.a. dadurch, dass die Tierwelt weniger Carbon freisetzt als die Pflanzenwelt mehr aufnimmt, so dass langfristig (all diese Betrachtungen sind langfristiger Art) der atmosphärische CO2-Gehalt wieder sinkt?”

    Tiere spielen absolut keine Rolle. Beim Uebergang von Eiszeit zu Warmzeit werden groessenordnungsmaeszig 1000 GTC in Biomasse gespeichert.
    EInfluss daher nicht gering (offensichtlich).
    Und (wie hier gezeigt) wenn das CO2 ungefaehr in Phase mit dem suedlichen Ozean ist, dann ist es eben ein Verstaerker, der von Beginn des Erwaermens an mitgewirkt hat. Dieses CO2 hat aber sicher viele Ursachen (suedlicher Ozean etc etc etc) und die Biosphaere wirkt dem genau entgegen. CO2 wird aus der Atmosphaere abgesogen und nicht umgekehrt freigesetzt.

  23. #23 axel
    März 9, 2013

    Und dass eine wärmere Atmosphäre mehr Wasserdampf besitzt, ist grundsätzlich nur “fragwürdig” und “hypothetisch”, wenn CO2 die Erwärmung verursacht hat, bei Erwärmung durch Zunahme der solaren Aktivität ist das ok. Oder hat jemand schon mal den Vorwurf gehört, das IPCC überschätzt die solare Aktivität?

  24. #24 Klimarealist
    März 10, 2013

    @Vennecke

    dem sattsam bekannten Verein verlogener Tattergreise

    Wenn ich mir Ihre generelle Wortwahl, Ihren Satzbau so betrachte, haben Sie Ihre schulische Laufbahn in den 50iger – 60iger Jahren auf dem Niveau Dorfschule begonnen und mit dem Abi, wenn überhaupt, in der nächstgrößeren Kreisstadt beendet, somit sind Sie in Sachen Alter breits zwischen Scheintot und Verwesung – klar dass Sie mit so Revoluzzer . Rentnern wie es sie bei EIKE so zu geben scheinz so Ihre Probleme haben und Sie da primär von einer ordentlichen Portion Neid geleitet werden 😀
    Oder etwa nicht ?
    😀

  25. #25 Günther Vennecke
    März 10, 2013

    Nun gibt man dem Klima”realisten” wieder einmal eine reelle Chance sich sachlich zu äußern und was kommt heraus?

    Das übliche, unflätige Gekeife.

    Ich habe doch begründet – wenn auch mit deutlichen Worten, aber bekanntlich gehört auf einen groben Klotz auch ein grober Keil – die unerhörte Verlogenheit im Dunstkreis von eike deutlich gemacht. (Wem dies noch nicht reicht, der kann sich ja mal die neuesten Eskapaden von Baron Monckton-Münchhausen zu Gemüte führen.)

    Aber Klima”realist” fällt wieder nichts anderes ein, als mit schrillen Tönen den Überbringer der für ihn peinlichen Nachricht zu schmähen – nicht, dass ich darauf etwas gäbe, denn was stört es die deutsche Eiche, wenn ein Borstenvieh (oder in diesem Fall ein Federvieh) sich an ihr rubbelt? – anstatt endlich einmal zuzugeben, dass er sich die falschen Freunde ausgesucht hat und einer Bande von Schwindlern und Ganoven aufgesessen ist.

    Die betreffenden Seilschaften der Klimaleugner sind seit Langem bekannt, nur Klimaleugner leugnen diese bisweilen noch.

    Die Reaktion von Klima”realist” auf mein Posting kann man allerdings auch so deuten: Getroffener Hund bellt (und die Karawane zieht ungerührt weiter). 🙂 🙂 🙂

  26. #26 Klimarealist
    März 10, 2013

    @Vennecke
    Sie missverstehen ich amüsiere mich über Sie – von “bellen” kann nun mal keine Rede sein, auch wenn Sie das gerne so sehen würden. 😀
    Sie betteln mal wieder um einen Kipp-Punkt – er Sei Ihnen verliehen, für dumm-dreistes Missverstehen.
    btw – wo sind denn Ihre “sachlichen”
    Äußerungen ?

  27. #27 Günther Vennecke
    März 10, 2013

    Klima”realist” scheint ein praktizierender Analphabet zu sein. Ich äußere mich zu einer Sache, nämlich dem Zusammenhang zwischen Energielobby und diversen “Vereinen” und Einzelpersonen, die im Internet auftreten und die Bevölkerung mit klimaleugnerischen Verdrehungen für dumm verkaufen wollen und er begreift das einfach nicht.

    Bestenfalls will er sich vor der für ihn unangenehmen Erkenntnis drücken und tut nur so, als würde er nichts begreifen. Aber der Grad dazwischen ist bei Klimaleugern ja in aller Regel sehr schmal. 🙂 🙂 🙂

  28. #28 Klimarealist
    März 10, 2013

    @GHoffmann
    Wier wollen mal festhalten, dass die Eisbohrkern-Saten von “Euch” “Alarmisten grundsätzlich in der Absicht hoch gehalten wurden, um die CO2 Messwerte zu rechtfertigen und musstet damit einhergehend in Kauf nehmen, den Nachlauf von 800 mit zu akzeptieren ( GH: ohne ck)
    Die Problematik der CO2 Messungen in Eisbohrkernen kann man hier ein wenig nachlesen
    Über Eisbohrkerne steht auch hier am Anfang einiges drin

  29. #29 Klimarealist
    März 10, 2013

    @Vennecke

    Ich äußere mich zu einer Sache, nämlich dem Zusammenhang zwischen Energielobby und diversen “Vereinen” und Einzelpersonen, die im Internet auftreten und die Bevölkerung mit klimaleugnerischen Verdrehungen für dumm verkaufen wollen

    Ich habe nach “sachlich” gefragt 😀
    War wohl nichts, nur Ihr übliches Geblubber.

  30. #30 Klimarealist
    März 10, 2013

    @GHoffmann
    bitte Mail mit 2 sachlichen Links aus dem Filter holen, danke

  31. #31 Georg Hoffmann
    März 10, 2013

    @Krishna

    Ich hatte bei skeptikal science eine nette Diskussion, wie denn nun die Klimaskeptiker so ticken. Ich sprach von !/3 Klimatotalverweigerer, 1/3 Verschwoerungstheoretiker und 1/3 ein-bisschen-physik-kann-nicht-schaden. Jaworozski ist das erste Drittel. Es gibt mittlerweile dutzende von Eis- und Firnmessungen und immer stimmen die CO2 Messungen ueberein. Unterschiedliche Tempperaturen unterschiedliche Akkumulation, unterschiedlicher Staubgehalt und immer zeigen die Messungen das Gleiche. So einen Mist wie Jaworowski, der das letzte Mal vor dreissig Jahren mal was am Eis gemessen hat, diskutier ich zumindest nicht mehr. Zu den Firnmessungen siehe die beiden Primaklima-Links oben.

  32. #32 axel
    März 10, 2013

    Ja, bitte, hol die Mail aus dem Spamfilter. Wir sind alle neugierig, ob Krishna tatsächlich zum ersten Mal hier im Thread etwas zum Thema zu sagen hat. Und er hätte ja viel zu sagen, er war ja auch stets ein Anhänger des “CO2 folgt der Temperatur”-Mantras.

    PS:
    “Beck ist zwar “nur” Biologielehrer, Diplombiologe und Biochemiker und hat nicht von ihm gemessene CO2 Werte ausgewertet, die so manchem evtl. nicht passen.
    Fest steht, daß für die bodennahen Aktivitäten des CO2 ( Senke etc ) die Mauna Loa Messwerte null Aussagekraft haben, die Auswertungen des Herrn Beck aber schon.”

    Krishna war halt schon 2008 Experte für CO2.

  33. #33 Georg Hoffmann
    März 10, 2013

    @axel
    Und jetzt weiss er sogar was an dem Porenabschluss so problematisch ist und kennt sogar den Jaworowski, Jahrgang 1927 (sagt das polnische wiki) und golden horseshoe Traeger.
    Physiktotalverweigerung.

  34. #34 axel
    März 10, 2013

    Na bitte, in der NoBrainZone wurde das Paper registriert.

    Nach einem Blick mit der berühmten Lupe auf die älteren Graphiken folgt dann messerscharf:

    “Also note the lag. The same is clear for the other interglacials. Clearly the scientists of the Parrenin et al Paper made some gross scientific errors, or the study is just a sham.”

    Außerdem hat sich ein deutscher Wettermann (Thüne) negativ über das Paper geäußert.

    Na dann: No brain, no pain.

  35. #35 axel
    März 10, 2013

    Und wo hat Gosselin die Graphik basierend auf Petit et al. 1999 her? Natürlich von der Seite von Ernst Georg Beck, die dessen Tod schadlos überstanden hat. Und da schließt sich der Kreis doch wieder sehr schön.

  36. #36 Georg Hoffmann
    März 10, 2013

    Pierre Gosselin zitiert Thuene, der das Paper natürlich durchschaut hat.
    Immer dieses Herangezoome. Die glauben wirklich und ernsthaft, dass das was mit Zoomen zu tun hat. Mein Gott, was fue Spackos.

    Geht es eigentlich noch absurder?
    Bestimmt. Luening zitiert Krishna der den rüstigen und gerade mal 85 jährigen Jaworowski zitiert. Dann stellen sich alle in eine Reihe und singen ein Rex Gildo Potpourri.

  37. #37 Thomas
    März 10, 2013

    @hoffmann
    Ich habe jene Diskussion zwar nicht mit verfolgt, aber ich teile Klimaleugner ebenfalls in 3 Kategorien ein:
    1: Spinner, die jeder noch so abstrusen Vereschwörungstheorie anhängen
    2. Bezahlte Lobbyisten der Energiekonzerne
    3. Gehören zu keiner der 2 Kategorien. Motiv rätselhaft.

  38. #38 axel
    März 10, 2013

    Jetzt geht’s hier auch schon los. Jaworowski ist seit knapp eineinhalb Jahren tot, alle meine verzweifelten Versuche, dies EIKE nahezubringen waren erfolglos, er ist immer noch als aktives Fachbeiratsmitglied gelistet.
    Tot, tot, tot, mausetot. Ist das denn alles so schwer??

  39. #39 Georg Hoffmann
    März 10, 2013

    @Axel
    Es war das Polnische Wiki. Ist ja doch ne komplzierte Sprache, wenn man so aus dem Romanischen kommt.
    Tot also. Wer haette das erwartet.

  40. #40 Georg Hoffmann
    März 11, 2013

    Ich habe bei Pierre Gosselin gefragt: How do you know the absolute dating of both records, ie Temperature and CO2?

    Auf Grund des offensichtlich beleidigenden Charakters dieser Frage wurde sie natuerlich geloescht.

  41. #41 Georg Hoffmann
    März 11, 2013

    Oh, er oder das System hat es sich anders ueberlegt.

  42. #42 E-O
    März 11, 2013

    @Thomas #37

    Selber denken….ist rätselhaft?, Propaganda anzweifeln….ist rätselhaft?……….bemerken, das die Energiewende ein Desaster ist….ist rätselhaft?……….jaja, wo der Geist fehlt ist alles rätselhaft, deswegen glaubt man ja auch jedem Guru, der ungestraft auch noch den dümmsten Kokolores an den Mann/Frau/Homo bringen will.

    Ach übrigens….Punkt 2 wird langsam irrelevant, weil die Energiekonzerne inzwischen auch auf grünen Subventionsstrom umgeschwenkt sind……..bleiben am Ende nur noch Spinner wie icke, man is dat Kacke, wa?

    PS…Ick hab weder ein Atomkraftwerk noch eine Kohledreckschleuder…..das einzige was ich im Blick hab, sind die Energiepreise, die unsere Kunden blechen müssen und die gerade durch die Decke gehen.

    E-O

  43. #43 Klimarealist
    März 11, 2013

    . “Synchronously” is defined by the scientists in this study as time differences of less than 200 years during the four known abrupt changes between both climate variables in the period examined.

    Nun, und wer ist nun vorne weg ?
    😀
    Für geologische Zeiträume ist das schon “synchron”, aber in der Logik, was folgt wem eben nicht.

  44. #44 Günther Vennecke
    März 11, 2013

    @E-O,

    “das einzige was ich im Blick hab, sind die Energiepreise, die unsere Kunden blechen müssen und die gerade durch die Decke gehen.”

    Ich denke, wenn ich bei Ihnen Kunde wäre, würde ich auch durch die Decke gehen. 🙂

  45. #45 Georg Hoffmann
    März 11, 2013

    @krishna

    1) Lesen sie den Artikel

    2) Wir kennen praktisch keinen Mechanismus der Milankovitch auf den Kohlenstoff wirken lassen wurde. Der Ocean erwärmt sich , das Seereise schwindet und (nach diesem Artikel) schon geht das Ausgasen los.

  46. #46 axel
    März 11, 2013

    Tja, schon das alte Paper zeigte die Probleme von Skeptikern im Umgang mit Unsicherheiten. CO2 folgt 800 Jahre, zack fertig, das ist halt so. Die MWP ist wärmer als alles andere heute, ist so, kein Zweifel, keine Unsicherheiten.

    Nein, im wahren Leben muss man mit Unsicherheiten umgehen können, auch hier gibt es keine einfachen schwarz-weiß Antworten.

    Zitat aus dem Paper:

    “We infer the aCO2-AT phasing at the four break points using a Monte-Carlo algorithm(supplementary
    materials): the onset of TI (10 +/- 160 years, 1s [s ist sigma], aCO2 leads), the onset of the Bølling oscillation
    (–260 +/- 130 years, AT leads), the onset of the Younger Dryas (60 +/- 120 years, aCO2 leads), and the onset of the Holocene (–500 +/- 90 years, AT leads). The uncertainty takes into account the uncertainty in the determination of the break points and the uncertainty in the determination of Ddepth. The only significant aCO2-AT lags are observed at the onsets of the Bølling oscillation and the Holocene. It should be noted that during these two events, the associated sharp increases in aCO2 were probably larger and more abrupt than the signals recorded in the ice core, due to the diffusion in the gas recording process (17).
    This atmosphere–ice core difference biases our break point determination toward younger ages. If we use these fast increases to determine the break points in aCO2, we find a lag of –10 +/- 130 years (1s) for the Bølling onset and –130 +/- 90 years (1s) for the Holocene onset; that is, no significant phasing.”

  47. #47 axel
    März 11, 2013

    Wo kam das CO2 her? Sehr wahrscheinlich aus den südlichen Ozeanen. Hypothesen und Simulation findet man z.B. hier: https://www.clim-past.net/8/149/2012/cp-8-149-2012.pdf

    PS:
    Bei WUWT ist man immer noch mit Widerlegungen des “Holozän-Hockeysticks” von Marcott et al. beschäftigt. Don “die Abkühlung kommt” Easterbrook hat eine mehrteilige Serie angekündigt. Teil 1: Die globale Rekonstruktion ist falsch, weil der grönländische GISP-Bohrkern etwas anders aussieht. Aha, alles was nicht lokal wie GISP aussieht, kann nie im Leben global sein, der Herr werfe Hirn vom Himmel (oh, hat er bereits getan, er hat’s nur nicht gleichmäßig verteilt 😉 )

  48. #48 axel
    März 11, 2013

    Apropos Carbon-Zyklus:

    Wurde uns nicht mal vor langer Zeit “Habemus nature Teil II” versprochen?

  49. #49 Georg Hoffmann
    März 11, 2013

    @axel
    Arrghh
    Nun gibts stattdessen Habemus Nature Teil III
    Zur temperaturentwicklung in der Eestantarktis. liegt aber noch unter Embargo.

  50. #50 E-O
    März 11, 2013

    @Günni
    Nun ja, wenn ein erboster Kunde bei uns auftaucht, erkläre ich ihn seine Rechnung……6,2 Cent für EEG, knapp 2 Cent für Netzausbau für die Energiewende und die restlichen Kleinigkeiten………und am Ende sage ich ihm, das er doch auch die Energiewende haben wollte………hihi, genau in dieser Sekunde sollte kein Grüner in der Nähe stehen.

    Ach übrigens, weißt Du was “der letzte Kunde” wirklich heißt?
    Nicht?
    OK….guck in den Spiegel.

    Ups
    E-O

  51. #51 E-O
    März 11, 2013

    @Georg

    Hmm, liegen nun CO2 und Temperaturen genau zeitgleich…..oder gibbet eine Unsicherheit in x Jahren?

    Läuft nun CO2 voraus…oder doch die Temperatur….800 Jahre waren eh etwas unlogisch….aber 5-10 Jahre würden sogar Sinn machen.
    Kopfkratz…….E-O

  52. #52 axel
    März 11, 2013

    @ Günni, E-O

    Seid ihr sicher, dass euer Thema zu Eisbohrkernen passt? Falls ja, schon erstaunlich, was man da so alles rauslesen kann.

  53. #53 E-O
    März 11, 2013

    @Axel-Schweiß
    Nun ja, ick hatte auf Thomas geantwortet……..Definition “Skeptiker”……….kann ja keiner ahnen, dat Günni sich wieder in Gespräche der Erwachsenen reinhängt.
    Sorry……..ick bleib jetzt janz fein artig beim Thema, Pionierehrenwort.

    Günni im Haus, erspart Skeptiker und Laus.
    E-O

  54. #54 Georg Hoffmann
    März 11, 2013

    @Heinz
    Immer vorsichtig sein bei dem was man erwartet. Sicher erwartet man praktisch Instantenes Ausgasen… Aber
    Nach diesem Paper sind es jedenfalls Null Jahre und ein paar Dekaden.

  55. #55 E-O
    März 11, 2013

    @Georg #54

    Joo, dat hat mich auch gestört……”und ein paar Dekaden”…..gehts nich etwas genauer?

    Mit dem Firnis is mir schon klar…..die 800 Jahre Differenz waren mir immer schon zu lang…..und zu unlogisch, wenn man gewisse schnelle Temperaturanstiege oder Abfälle berücksichtigt.

    Kann man eine generelle Aussage machen…..ob erst die Temperatur stieg und dann das CO2……oder umgekehrt…….oder sind die Schichtungen der Eiskerne dazu nicht aussagefähig genug?

    MfG
    Onkel Heinz

  56. #56 Alfred Wegener
    März 11, 2013

    Verstehe ich das richtig, dass bei Ihnen der Siebenjährige Krieg und die Kuba-Krise gleichzeitig stattfanden?
    Zitat: “Synchronously” is defined by the scientists in this study as time differences of less than 200 years.

  57. #57 axel
    März 11, 2013

    Hallo Heinz,

    ich denke, die Frage “wer führte, wer folgte, und wieviel?” zu simplifizierend ist. Der Übergang bestand aus vielen Schritten, jeder Schritt hat seine eigenen Gesetze. Wann wurde welcher Carbonpool angezapft? Was war mit den Meeresströmungen? Besonders letzter Punkt ist bedeutsam, er kann dazu führen, dass sich die antarktische Erwärmung von der globalen entkoppeln kann (siehe Shakuns Vorschlag des “Schaukelmechanismus”).

    Wer führte wann um wie viele Jahre mag auch interessant sein. Das größte Rätsel ist wohl eher, wo das ganze CO2 herkam, simples Ausgasen durch wärmere Ozeane kann nach Georg (und anderen Wissenschaftlern) ja höchstens +30 ppm erklären.

    Spannend, mal schauen, ob Habemus Nature III einen weiterer Babyschritt vorwärts verheißt.

  58. #58 axel
    März 11, 2013

    @ Alfred Wegener

    Das müssen Sie schon mit sich selbst klären. Die Presseerklärung mit diesem Wortlaut stammt vom Alfred Wegener Institut 😉

  59. #59 Georg Hoffmann
    März 11, 2013

    @alfred
    “Verstehe ich das richtig, dass bei Ihnen der Siebenjährige Krieg und die Kuba-Krise gleichzeitig stattfanden?
    Zitat: “Synchronously” is defined by the scientists in this study as time differences of less than 200 years.”

    Zitat von wem? Ich sehe keine 200 Jahre in der Studie.
    Der Beginn de Bollinger/allerod ist etwa mit einem Delay von -10 +- 130 Jahre angegeben.

    Und das ist natürlich fast ein wissenschaftliches Wunder, dass man so etwas vor 12000 Jahren mit solch unglaublicher Praezision angeben kann.

  60. #60 E-O
    März 11, 2013

    @Axel-Haar-Rasierer

    Tja…..und da sind wir wieder bei Hänsel und Gretel……wer hat die Brotkrumen aufgemampft?

    Wenn CO2 die Erde überhitzt….muss sie automatisch führen…..wenigstens bis Stalingrad……..ansonsten wäre eure These des AGW ja nicht beweiskräftig….oder aber, ohne Onkel Georg verärgern zu wollen,……spielen euch natürliche Faktoren einen Streich?

    MfG
    Onkel Heinz

  61. #61 Georg Hoffmann
    März 11, 2013

    @heinz
    Ich hab keine Ahnung wovon da sprichst.
    Versteh diesen Blogg auch als die fantastische Gelegenheit Infos gratis auf deine ganz eigenen Fragen zu bekommen.

  62. #62 E-O
    März 11, 2013

    @Georg
    Ups….ich vergass, dat Wissenschaftler mit meiner Ausdrucksweise immer etwas Probleme haben……..da ick aber jetzt echt zu müde bin und meine Birne schon abseits von gut und böse ist, versuch ich dat mal morgen etwas genauer zu definieren.

    Dat Fleisch is willig….nur der Geist ist zu schwach.
    MfG
    Onkel Heinz

  63. #63 axel
    März 11, 2013

    @ E-O

    Das Faszinierende ist doch, wie so ein kleiner Trigger (Milankovic-Zyklen), nur ein bisschen Umverteilung von solarer Strahlung auf dem Globus, wie dies zu einem, völlig anderen Klimazustand mit 4-6° Unterschied führen kann.

    Diese Mechanismen aufzuschlüsseln, das ist spannend. Kein Mensch käme heute noch auf die Idee, damit zu “beweisen”, dass CO2 klimawirksam ist. Mit Verlaub, der Zug ist schon lange abgefahren, und wer es nicht glaubt, der soll einfach mal einen Mechanismus ohne CO2-Wirkung vorschlagen und durchrechnen (da schweigen die Vahrenholts und die anderen Kasper plötzlich ganz kleinlaut).

    Nee, man benutzt die Daten, um aus der Veränderung (Glazial zu vorindustriell heute) die Klimasensitivität zu messen. Da nur der Anfangs- und Endzustand eingeht, ist es egal, wie die Lags denn nun tatsächlich sind. Am häufigsten liest man von Ergebnissen für die Klimasensitivität im Bereich von 2 – 3,5°C, Annan tippte letztens auf um 2,5° herum.

    PS:
    Probleme haben höchsten die, die meinten, der (scheinbare) Lag von 800 Jahren beweise, dass CO2 keine Rolle spielt. Wenn nun CO2 führt, was “beweist” dies denn dann nach der Logik ebendieser Leute? Hm, das muss man dann erst mal seinen Anhängern irgendwie vermitteln, bislang beschränkt man sich auf Schweigen bei WUWT und seinen Followern.

  64. #64 axel
    März 11, 2013

    Die 2. “Widerlegung” von Marcott et al. bei WUWT. Erinnert an Shakun et al. 2012, damals habe ich nach der 6. “Widerlegung” mit dem Zählen aufgehört.

    Die Anzahl der Widerlegungen ist ein guter Proxy dafür, wie stark ein Paper ins Skeptikermark getroffen hat.

  65. #65 E-O
    März 12, 2013

    @Georg
    Ok, neuer Versuch.
    Ganz einfache Frage, da ich dat als Laie irgendwie nicht richtig schnalle……..kann man einen prinzipiellen Trend erkennen, ob nun CO2 vorneweg rennt oder die Temperatur oder ob das bei den Unsicherheiten immer noch nicht richtig erkennbar ist.
    Andere Frage….habt Ihr in Grönland auf Höhe der Gletscher eine CO2 Messung….die schon ein paar Jahre mitläuft?
    MfG
    Onkel Heinz

  66. #66 E-O
    März 12, 2013

    @Axel-VerKohler
    Trigger sind ja wat feines…aber ick hab mal im (von Warmies kontrollierten) Wikepedia nachgeguckt und da steht…
    “”””Auf der Erde werden die Zyklen sichtbar als periodisch auftretende Abweichungen der Solarkonstante in der Größenordnung von circa 5 bis 10 Prozent. Als planetare Ursache für diese Schwankungen der Intensität der Sonneneinstrahlung auf der Erde wurden die folgenden drei sich überlagernden Effekte berechnet:

    Die Präzession der Erdrotationsachse („trudelnder Kreisel“) mit Zyklen von 25.700 bis 25.800 Jahren
    Die Erdschiefe (Veränderung des Neigungswinkels der Erdachse) mit einem Zyklus von 41.000 Jahren
    Die Änderung der Exzentrizität (Variation des Radius der Erdumlaufbahn um die Sonne) mit einem Zyklus von 100.000 Jahren.

    Dabei kommt es zwar nicht zu einer globalen Veränderung der Sonnenstrahlung im Jahresmittel, aber da die Landmasse der Erde sich hauptsächlich auf die Nordhalbkugel der Erde konzentriert, ändert sich die Verteilung der Sonnenstrahlung auf die Jahreszeiten u.U. erheblich. Dieser Effekt ist bedeutsam, da sich an Land Strahlungsänderungen erheblich schneller und stärker auf die Temperatur auswirken als über dem Meer.”””””

    Hmm, Größenordnungen von 5-10 % sind ja nun kein Pappenstiel, oder?
    Weiter in Teil 2
    E-O

  67. #67 E-O
    März 12, 2013

    @Axel
    Hmm, von wat sprechen wir hier eigentlich?
    Eine ungleichmäißg beschichtete, halbwegs gut isolierte Murmel, die von aussen beheizt wird.
    Folgende Störgrößen beeinflussen dat System.
    a) die Heizungs selber, schwankende Heizleistung der Sonne. (1%)
    b) die torkelnde Erde, je nach dem welche Seite beheizt wird (5-10%)
    c) Wasser……..und da muss man nochmals unterscheiden zwischen
    c1) Eis….verändert auf der Erdoberfläche und Wolken dat Albedo (Reflexion der eingestrahlten kurzwelligen Sonnenstrahlen)
    c2) Wasser…in Form unserer Ozeane ein richtig fetter Wärmespeicher, in Form von verdunstenden Wasser, ein Kühler der Erdoberfläche.
    c3) Wasserdampf…….1-4 Vol.Prozent in der Atmosphäre, mit wirklich fetten Absorbtionslinien über dat gesamten IR-Band
    ausserdem gibt Dampf beim Kondensieren Wärmeenergie an die Umgebung ab.
    d) CO2 (+sämtlichen anderen IR-aktiven Gasen) mit 0,04% Vol, Anteil an der Atmosphäre, dat einen schmalen Bereich (15µm samt Seitenbänder) absorbiert, in den unteren Schichten thermalisiert und wahrscheinlich bis 100m Höhe fast alles absorbiert hat.

    Für mich als Regelungstechniker sind immer die größten Einflussgrößen entscheidend, kleine Störgrößen fallen im Gesamtsystem kaum auf…….daher sagt mir die Logik, dat CO2 als mickrige Störgröße nur einen marginalen Einfluss aufs Klima-Gesamtsystem haben kann.
    So Axel……nun darfste mich peitschen, aber bitte nich mit Papers sondern mit eigenen Worten, denn mit dem Klima-Fachchinesisch kann ick ja eh kaum wat anfangen.
    MfG
    Onkel Heinz…….au Backe, gleich gehts tierisch ab hier

  68. #68 E-O
    März 12, 2013

    Nachtrag…..bevor einer meckert….
    e)Aerosole/Dreck in der Atmosphäre mit Mehrfachwirkung…..Wolkenbildung, Albedo und erwärmend….natürliche (Vulkane, Meere, Waldbrände, Staubstürme) und menschlliche (Kraftwerk ohne Staubfilter, Dieselruss u.v.m)
    f) Veränderung der Erdoberfläche durch den Menschen (Urbanisierung und dadurch Änderung des Albedos)
    und alles, was ich jetzt noch vergessen habe, mit aufzuzählen.
    E-O

  69. #69 Georg Hoffmann
    März 12, 2013

    @Heinz
    “Ganz einfache Frage, da ich dat als Laie irgendwie nicht richtig schnalle……..kann man einen prinzipiellen Trend erkennen, ob nun CO2 vorneweg rennt oder die Temperatur oder ob das bei den Unsicherheiten immer noch nicht richtig erkennbar ist.”

    VOR diesem Paper gab es mindestens zwei Paper die klar aussagten, dass das CO2 mit einigem Verzug nachhinkte (800±600 Jahre). Die Aussage gilt also sog innerhalb der sicher groszen Unsicherheiten.
    Mit diesem Paper kann man das praktisch nicht mehr sagen: 10 ±180 Jahre beisbielsweise beim Boelling/Allerod bedeutet praktisch gleichzeitig, wobei eben nicht mehr zwischen leichtem Vorlauf oder leichten Nachlauf unterschieden werden kann.
    UNABHAENGIG von all diesen Eiskern Papern gilt aber: Wir kennen keinen Mechanismus, der nur auf das CO2 (also den Kohlenstoffkreislauf) wirken wuerde und nicht auch immer die Temperaturen beeinflusste. Ralph Keeling etwa schlu einmal vor, dass groessere Mengen CO2 unter dem glazialen Meereismassen schlummerten, die schnell mobilisiert wuerden, sobald das Meereis verschwindet. OK als Hypothese aber auch da ist es natuerlich so, dass durch das Verschwinden des Meereis eben eine Erwaermung der Ozeanoberflaeche erfolgt und somit doch wieder CO2 und T praktisch gleichzeitig ansteigen wuerden.
    Kurz basierend auf Ueberlegungen unabhaengig von diesen Eiskern Papern meine ich, dass das CO2 immer der Temperatur leicht nachhinken muss. Das neue Paper sagt, dass das Nachinken fast null betraegt. Vorher war von mehreren 100ert Jahren die Rede. Vorlaufen kann es in den Milankovitch Eiszeit Zyklen praktisch nicht.
    Sucht man erdgeschichtliche Belege dafuer, dass auch mal das CO2 vorlaeuft, muss man etwa sich das PETM anschauen (riesige Mengen aus dem Ozean ausgasendes Methan, weches natuerlich schnell in CO2 in der Atmosphaere umgesetzt wird)
    https://en.wikipedia.org/wiki/Paleocene%E2%80%93Eocene_Thermal_Maximum

    oder Phasen intensiven Vulkanismus.

  70. #70 Georg Hoffmann
    März 12, 2013

    @Heinz
    “Auf der Erde werden die Zyklen sichtbar als periodisch auftretende Abweichungen der Solarkonstante in der Größenordnung von circa 5 bis 10 Prozent. ” Das ist eine sehr schlechte Formulierung. Es gibt grosse Aenderungen in einem Breitenkreis zu einer Saison. Aber ueber das Jahr gemittelt ist es dan schon deutlich weniger. Ueber den Planeten gemittel ist es praktisch null (es bleibt nur ein winziger Beitrag, der mit der Exzentrizitaet geht). All das hat so oder so nichts mit der Solarkonstante zu tun. Kannst du mal den Wiki Link setzen?

  71. #71 E-O
    März 12, 2013

    @Georg
    Erstmal Danke für die Mühe, die Du Dir mit mir alten Sack gibst.
    Hier mal der Wiki-Link.
    https://de.wikipedia.org/wiki/Milankovi%C4%87-Zyklen
    Anonsten lass ich mir gerade Deine Ausführungen auf der Zunge zergehen, bzw durch die Omme gehen, weshalb ick da erst später antworten möchte, die alte Betriebssoftware läuft da leider etwas zu langsam.

    Ähhmm, trotzdem noch ne Frage, ist die folgende Abbildung soweit in Ordnung für Dich als Klimaforscher?
    https://commons.wikimedia.org/wiki/File:Ice_Age_Temperature.png

    MfG
    Onkel Heinz

  72. #72 E-O
    März 12, 2013

    Mist…bei Euch wird wohl schon ab den 2. Link moderiert?
    E-O

  73. #73 axel
    März 12, 2013

    Heinz,

    darf ich? Die Temperaturen zeigen die EPICA-Temperaturen, die auf keinen Fall mit den globalen Temperaturen verwechselt werden sollten. Hörte man früher häufig von Temperaturunterschieden von ca. 6°C zwischen Glazial und Interglazial, heute scheint man eher von um die 4°C auszugehen.

    Die 5-10% sind Blödsinn. Interessant, das erste Mal, dass ich bei Wikipedia so richtigen Blödsinn finde. Überlege selbst:

    Neigung der Erdachse, Präzession: Ändert die einfallende solare Strahlung überhaupt nicht, immer noch derselbe Querschnitt, auf den die Strahlung trifft. Ein Effekt ergibt sich nur aus der unterschiedlichen Landverteilung zwischen Nord- und Südhemisphäre, aus jahreszeitlichen und räumlichen Umverteilung der Strahlungsenergie und schließlich aus dem Eisalbedofeedback.

    Ob man bei den 5-10% lokale Änderungen meint? Oder Änderungen der Albedo mit einbezogen hat? Keine Ahnung, jedenfalls ist diese Zahl blödsinnig oder grob missverständlich.

    Zum Vergleich: Eine Änderung der solaren Aktivität um 3% (ich hoffe, ich hab’s richtig im Kopf) entspricht einer Verdopplung von CO2.

    PS Re #67:
    Ich verstehe das Problem nicht. Änderung der TSI, Einfluss der Änderung der Eisalbedo. Kann man alles leicht abschätzen. Trotzdem fehlt halt noch ein ganzes Stück (ganz grob: die Hälfte), um die tatsächliche Erwärmung zu erklären. CO2 erklärt alles ganz leicht, die einzig spannende Frage ist: wo kam es her? Auch wenn diese Frage für die Temperaturentwicklung bis 2100 völlig belanglos ist (nicht aber für längerfristigere Prognosen), wir sind einfach neugierig, Heinz.

  74. #74 Wolfgang Flamme
    März 12, 2013

    Die Sachlage erinnert mich an den alten jiddischen Witz, wo sich der Schmul beim Itzig beschwert, daß in dem Krug, der er ihm ausgeliehen hatte, ein Loch ist. Entgegnet der Itzig: “Also erstens habe ich mir von Dir gar keinen Krug ausgeliehen. Zweitens war das Loch schon drin, als ich ihn mir ausgeliehen hab. Und drittens ist in dem Krug gar kein Loch.”

  75. #75 Günther Vennecke
    März 12, 2013

    @axel,

    “Eine Änderung der solaren Aktivität um 3% (ich hoffe, ich hab’s richtig im Kopf) entspricht einer Verdopplung von CO2.”

    Sorry, aber das kann nicht richtig sein. Bestenfalls kann ein Anstieg der solaren Aktivität um 3% (aber wann gibt es so etwas schon?) zu einer CO2-Verdoppelung führen. Ein Absinken der solaren Aktivität dürfte dann entsprechend durch den damit verbundenen Temperaturrückgang zu einem Rückgang der CO2-Konzentration führen.

  76. #76 Klimarealist
    März 12, 2013

    Neigung der Erdachse, Präzession: Ändert die einfallende solare Strahlung überhaupt nicht, immer noch derselbe Querschnitt, auf den die Strahlung trifft.

    Ist schon ein Unterschied, in welchem Winkel die Sonne auf die Erde trifft.
    Und nochmal, TSI ist nicht alles, nicht das, worauf es ankommt.

  77. #77 axel
    März 12, 2013

    *facepalm*

    Du bist selten dämlich, Krishna.

  78. #78 axel
    März 12, 2013

    @ Günther

    Gemeint war.
    Eine Zunahme der TSI um rund 2% (3% war falsch) hat dieselbe Erwärmungswirkung wie eine Verdopplung der CO2-Konzentration.

  79. #79 Georg Hoffmann
    März 12, 2013

    @krishna
    “Ist schon ein Unterschied, in welchem Winkel die Sonne auf die Erde trifft.”
    Ein Unterschied wofür?
    Die Schraege vertailt Emergie unterschiedlich zwischen den Breitemgraden. die Gesamtenergie ist exakt EXAKT gleich.
    “Und nochmal, TSI ist nicht alles, nicht das, worauf es ankommt.”
    Nur das die Intensitaet auch der UV Strahlung nicht von der Position der Erde beeinflusst wird. Somit weiß ich wirklich nicht was das fuer die Ilamkovitch Variationen bedeuten soll. ich sag mal, irrelevant.

  80. #80 axel
    März 12, 2013

    @ Krishna

    Falls du das immer noch nicht kapierst, hier einfacher:

    Wie ändert sich der auf eine Kugel auftreffende Energiefluss, wenn du die Kugel drehst?
    A) gar nicht
    B) hängt vom Auftreffwinkel ab
    C) bei Tag oder bei Nacht?
    D) bin gerade weg
    E) Die Erde ist keine Kugel.
    F) keine Ahnung

    Mehrfachantworten sind möglich.

    Tipp: Eine Kugel hat verdammt viele Symmetrieachsen.

  81. #81 E-O
    März 12, 2013

    @Axel
    Natürlich darfst Du, wir ham doch sowas wie friedliche Koexistenz…..d.h., frech wie Bolle, Spitzen werfen bis der Arzt heult aber nix unter der Gürtelinie, davon hab ick schon vom Heinzow genug.

    Tja Wikipedia…..ick dachte bisher immer, dat Ihr die Klimathemen dort voll im Griff habt, dat Du Dich jetzt über Wiki wunderst, wundert mich wiederum.
    Da sich Onkel Georg nicht zu den Linke geäussert hat, nehm ick mal an, das diese Temperaturrekonstruktion soweit in Ordnung ist.
    https://commons.wikimedia.org/wiki/File:Ice_Age_Temperature.png

    Wat fällt uns da auf?
    In den letzten 450 k Jahren waren dort 5 Temperaturanstiege, die genauso scharf noch oben stiegen wie jetzt im 20. Jahrhundert.
    So, Ihr versucht dat jetzt krampfhaft irgendwie dat CO2 in die Schuhe zu schieben, ersatzweise dem Methan, dat eine typische Verweilzeit in der Atmosphäre von 12 Jahren hat, aber nicht nur in CO2 zerfällt.
    So, Dank der neuen Studie, die Onkel Georg als Anlass dieses Artikels genommen hat, kann das CO2 gleichzeitig oder vor oder den Temperaturen nachlaufen, nu bin ick diebezüglich immer noch nicht schlauer geworden, ob historisch gesehen, dat CO2 überhaupt eine klimatische Auswirkung hatte.

    Och Menno….drüben nur Nieten gezogen und hier läuft es auch nicht viel besser.
    Ick werd wohl dumm sterben müssen.
    Onkel Heinz…….Nase voll für heut.

  82. #82 E-O
    März 12, 2013

    @Axel
    Nachtrag
    Wie Du sicherlich weißt, ist die Erde keine exakte Kugel, sondern an den Polen etwas platt aufgrund der Fliehkraft.
    Weiterhin kommt es auf die Oberflächenstruktur an…und da sind eben die Landmassen besonders auf der Nordhalbkugel konzentriert und die erwärmen sich ja bekanntlich viel schneller als die Ozeane.
    Ick gloobe, Du machst es Dir etwas zu einfach auf den Einwurf Krishna, mit Deiner Antwort.
    MfG
    Onkel Heinz….nu is aber wirklich Feierabend, gute Nacht

  83. #83 axel
    März 12, 2013

    @ Heinz

    “In den letzten 450 k Jahren waren dort 5 Temperaturanstiege, die genauso scharf noch oben stiegen wie jetzt im 20. Jahrhundert.”

    Obacht, ein Anstieg der globalen Temperatur um 4-6°C in einigen Jahrtausenden seit der letzten Eiszeit kann niemals genau “so scharf” sein wie ein Anstieg von 2-4°C in einem Jahrhundert, da ist in der Rate ein Unterschied in einer Größenordnung vorhanden.

    “So, Dank der neuen Studie, die Onkel Georg als Anlass dieses Artikels genommen hat, kann das CO2 gleichzeitig oder vor oder den Temperaturen nachlaufen, nu bin ick diebezüglich immer noch nicht schlauer geworden, ob historisch gesehen, dat CO2 überhaupt eine klimatische Auswirkung hatte.”

    Zum Vor- oder Nachlaufen. Mir erscheint es logisch, dass es anfangs nachlief, da Milankovic-Zyklen der Auslöser waren. Ich glaube auch nicht, dass es für die Zeiten danach eine einheitliche Antwort gibt, da spielen sich zu viele Prozesse parallel ab.

    Ob CO2 eine klimatische Auswirkung hat? Die Frage verstehe ich nicht, ich hatte ja schon erwähnt, dass niemand den Temperaturunterschied ohne CO2-Wirkung erklären kann, auch Skeptiker nicht.

    Du vermischst hier zwei Ebenen miteinander:

    Die erste Ebene ist die wissenschaftliche. Für die Wissenschaft ist die verbesserte zeitliche Datierung wichtig, weil es helfen könnte zu verstehen, welche Kohlenstoffpools wann angezapft worden sind.

    Die zweite Ebene ist die skeptische Perspektive. In vielen Skeptikerblogs wurde der Strohmann aufgebaut, CO2 könne keine Wirkung entfaltet haben, da es ja der Temperatur hinterherlief. Wissenschaftlich bedeutungslos, da gleich mehrfach blödsinnig, aber eben bedeutsam für Skeptiker. Auf diesen Randeffekt zielt Georg ab: Das blödsinnige Skeptikerargument fällt ersatzlos weg, falls Skeptiker sich aufraffen, wissenschaftliche Erkenntnis zu Kenntnis zu nehmen, auch wenn diese ihren Hoffnungen und Wünschen widerspricht.

    Du tust so, als wäre die Bedeutung des Papers auf der Ebene 2, sorry, aber das ist falsch. Ebene 1 ist spannend, ob CO2 klimawirksam ist, kann nur noch anachronistische Skeptikerseelen bewegen. Entschuldige meine offenen Worte, aber das wird ansonsten niemand auch nur diskutieren wollen. Wir stark es klimawirksam ist, das ist wiederum spannend.

    PS:
    Weiter oben hast du eine Reihe von Details angesprochen: Aerosole, Wasserdampffeedback etc. Dir entgeht eine Sache: Bei der Methode der Bestimmung der Klimasensitivität durch Vergleich von Glazial zum vorindustriellen Wert stecken in der Erwärmung ja schon alle Feedbacks drinne, anthropogene Aerosole gab es auch nicht. Man muss ja gerade nicht aufschlüsseln, welche Werte welche Feedbacks haben, man benutzt ja nur die Erwärmungswerte, die die Feedbacks schon beinhalten. Elegant, nicht wahr? Leider sind die Werte immer noch recht unsicher, liegt hauptsächlich daran, dass man die globale Erwärmung nicht genau genug kennt, da hilft die antarktische EPICA-Kurve nicht wirklich weiter (die Erwärmung ist an den Polen stärker als z.B. in den Tropen).

  84. #84 Georg Hoffmann
    März 12, 2013

    @heinz
    Hab mir die Grafik noch nicht angeschaut. Nur soviel:
    Die sicherlich superschnellen Temperaturschwankungen der Eiazeiten dauern ca 10000 Jahre. Momentan diskutieren wir 200 Jahre.
    Die Verteilung Der Landmasse ist exakt die Idee von Milamkovitch. darum ist fuer ihn auch die Einstrahling bei 60 Grad Nord entscheiden.
    Aber was am oberen Ende dér Atmosphaere ankommt, ist davon Voellig unabhängig. das ist ein rein geometrisches Poblem. Nochmals nur die Exzentrizität hat da einen sehr kleinen Einfluss. Die Schraege der Érdachse ist Wurscht fuer das globale Mittel (auch fuer ein elipsoid).
    Und in jedem Fall hat nichts von all dem irgendetwas mit dem UV Output der Sonne zu tun.
    Ich versuche das vielleicht mal in einem extra Posting in Ruhe zu erklären.

  85. #85 axel
    März 12, 2013

    @ Heinz (#82)

    Schrieb ich doch auch alles an dich, du erinnerst dich?

    Neigung der Erdachse, Präzession: Ändert die einfallende solare Strahlung überhaupt nicht, immer noch derselbe Querschnitt, auf den die Strahlung trifft. Ein Effekt ergibt sich nur aus der unterschiedlichen Landverteilung zwischen Nord- und Südhemisphäre, aus jahreszeitlichen und räumlichen Umverteilung der Strahlungsenergie und schließlich aus dem Eisalbedofeedback.

    Krishna zitiert nur den ersten Satz, dort beziehe ich mich auf die einfallende Strahlung und für diese ist das trivialerweise richtig. Der zweite Satz beschreibt Einflüsse, die die ausgehende Strahlung verändern, z.B.: weniger Eis, weniger reflektierte Strahlung.

    Krishna konzentriert sich hier hauptsächlich auf Beleidigungen, Beschimpfungen und dem Setzen von Links, deren Inhalten er allermeist erst gar nicht kommentiert oder interpretiert. Und wenn er ausnahmsweise mal inhaltlich selbst etwas sagt, ist es saudämlich. Ich bleibe daher bei der Aussage in #77

  86. #86 axel
    März 12, 2013

    @ Heinz

    Ein Argument noch vorm Schlafen 😉

    Skeptiker tun sich ja gemeinhin schwer mit der Rolle der Feedbacks. Hätte CO2 aber keine Wirkung gehabt bei der Erwärmung seit dem letzten Glazial, dann wäre das Forcing viel geringer und würde praktische nur noch auf das Forcing durch Veränderung der Eis-Albedo beschränkt werden. Wenn ein kleineres Gesamtforcing aber die Erwärmung bewirkt haben sollte, dann müssten die Feedbacks mal ganz grob etwa doppelt so stark sein als bislang gedacht. Die Klimasensitivität (die ja unabhängig von der Art des Forcings ist, “CO2-Klimasensitivität” ist ein unglücklicher Name, der nur die Stärke des Forcings beschreiben soll) wäre etwa doppelt so groß.

    Ganz schön vertrackt aus Skeptikersicht, nicht wahr? Je weniger CO2-Wirkung, um so größer der Effekt der Feedbacks. Es überrascht nicht, dass Skeptiker diesen Aspekt noch nie diskutiert haben.

  87. #87 E-O
    März 13, 2013

    @Axel-Schlaumeier
    Erstmal vorab…..Du bist hier nicht bei EIKE.
    Ick bin hier alleine und diskutiere Probleme, die nur mir durch den Kopf gehen, geschuldet des Versuches zu einer Erkenntnis zu kommen, die mir hilft die Klimaproblematik zu verstehen.
    Ich regestriere zwar die unterschiedlichsten Meinung beider Seiten, lasse diese gelegentlich auch in meine Überlegungen einfließen, wenn sie mir persönlich logisch erscheinen, lasse mich aber davon nicht zu sehr beeinflussen.

    Ergo……vergiss die Skeptikerszene und antworte mir so, als müsstest Du es einem Kind erklären, akzeptiere, dat ick ick bin und sonst nix anderes….und in mir wehrt sich jede Faser des Körpers dagegen, das eine kleine Störgröße eine weit aus mächtiger Störgröße vor sich hertreiben soll.
    Dat aber…..ist meine ganz eigene Meinung, die Du nur mir ausreden musst und sonst niemand anderem

    Guck mal, Onkel Georg hat dat geschnallt, also kannst Du dat auch……wenn ick mit Fanatikern reden will, kann ick mir auch Günni oder Heinzow vornehmen, wozu ick aber keinen Bock mehr hab, weil das reine Zeitverschwendung ist.
    Ok, ick hoffe, dat ist jetzt endlich mal geklärt……
    MfG
    E-O

  88. #88 E-O
    März 13, 2013

    @Axel-Haxe mit Senf
    Zitat Deinereiner:
    “””Obacht, ein Anstieg der globalen Temperatur um 4-6°C in einigen Jahrtausenden seit der letzten Eiszeit kann niemals genau “so scharf” sein wie ein Anstieg von 2-4°C in einem Jahrhundert, da ist in der Rate ein Unterschied in einer Größenordnung vorhanden.”””

    Obacht mein Freund……Wir haben, aus einer kleinen Eiszeit kommend, in 153 Jahren eine Temperaturerhöhung von zirka 0,7 Grad messtechnisch halbwegs nachweisbar, erlebt. Wenn ick fies wäre, würde ich die wärmere Periode vor der kleinen Eiszeit als Masstab ansetzen und würde dann auf eine Erwärmung von popelige 0,3-0,4 Grad Erwärmung kommen. Deine 2-4 Grad kommen aus dem Computer…..sind also nicht eingetroffen und es ist inzwischen fraglich, ob sie überhaupt eintreffen werden.

    CO2……ick hab nie geleugnet, das dieses Gas in der Lage ist, in schmalen Frequenzbändern IR-Strahlung zu absorbieren, ick bin nur nicht damit einverstanden, das ihr dem CO2 solch katastrophale Klimawirksamkeit unterjubeln wollt.
    Mir ist es nämlich so ziemlich wurscht, ob die CO2-Konzentration nun 800 Jahre, oder nur 10 Jahre hinterherhumpelt oder sogar gleichzeitig ansteigt……..wichtiger wäre die Aussage gewesen, das Co2 dem Temperaturanstieg signifikant vorauseilt……tja, und dat lässt die Studie leider immer noch offen.
    Dat sich Georg in diesen Zusammenhang neugierig fragt, woher nun dat CO2 kommt, dat zu einer derartigen Anhebung der Konzentration führt…..ist seinem wissenschaftlichen Wissensdurst geschuldet…..und seine Ergebnisse dahingehend, würden mich auch interessieren.
    MfG
    E-O

  89. #89 M.Schmitz
    März 13, 2013

    “und in mir wehrt sich jede Faser des Körpers dagegen, das eine kleine Störgröße eine weit aus mächtiger(e) Störgröße vor sich hertreiben soll.”
    E-O :
    Ich denke was eine subjektiv empfundene “mächtige Störgrösse” für den Lebensraum des Menschen darstellt sieht von aussen betrachtet eher ziemlich mickrig aus .

    Ich würde eher so formulieren: eine winzige Stellgrösse bedingt eine kleine Variation im Klimageschehen welche aber ausreicht den schmalen Korridor des Lebens nachhaltig zu stören .
    Ich denke man sollte praktischerweise subj. Empfindungen bei der Bewertung von Stellgrössen misstrauisch gegenüberstehen.
    mfg,

  90. #90 E-O
    März 13, 2013

    @M.Schmitz
    Nun ja, wenn man unseren Planeten als lebenden Organismus sieht, könnte diese Ansicht ja ganz praktisch sein (Arsen und tote 80 Kg Fleisch)…..für mich ist dat Dingens aber nichts weiter als ein Brocken Fels im All, der dat Glück hat im richtigen Abstand zur Sonne zu stehen und dazu noch jede Menge Wasser rumschwimmen hat.
    Ohne Wasser…..hätte dieser Planet ein völlig anderes Klima…..und ohne Wasser hättet ihr keine Feedbacks, die ihr dann als Grund der eigentlichen Überhitzung dem Volke unterjubeln könntet.

    Aber genau diese Feedbacks…….scheinen euch derzeit einen kleinen Strich durch die Glaskugel zu machen.
    Subj. Empfindungen sind es nur scheinbar…..es ist das Unterbewusstsein, das aus Lebenserfahrung + erlernten Wissen ein Warnsignal ausstößt, es liegt nun am Bewusstsein, diesem eine fundierte Stimme zu geben……und da hakt es bei mir derzeit noch a bisserl.
    Ick sag mal so, ein paar Seitenbänder, die sich nun bei zunehmender CO2 Konzentration anschicken etwas Wirkung zu zeigen, hauen mich energetisch nicht wirklich aus den Socken….fallst de weißt, wat ick meine.
    E-O

  91. #91 markus
    März 13, 2013

    Da sich Onkel Georg nicht zu den Linke geäussert hat, nehm ick mal an, das diese Temperaturrekonstruktion soweit in Ordnung ist.
    https://commons.wikimedia.org/wiki/File:Ice_Age_Temperature.png

    Wat fällt uns da auf?
    In den letzten 450 k Jahren waren dort 5 Temperaturanstiege, die genauso scharf noch oben stiegen wie jetzt im 20. Jahrhundert.

    Wir haben, aus einer kleinen Eiszeit kommend, in 153 Jahren eine Temperaturerhöhung von zirka 0,7 Grad messtechnisch halbwegs nachweisbar, erlebt. Wenn ick fies wäre, würde ich die wärmere Periode vor der kleinen Eiszeit als Masstab ansetzen und würde dann auf eine Erwärmung von popelige 0,3-0,4 Grad Erwärmung

    Ob die Temperaturrekonstruktion soweit in Ordnung ist oder nicht es bleibt eine Rastergrafik mit der Breite 564px
    Selbst wen man die Beschriftung ignoriert wären die 150 Jahre nur 1/5px breit also nicht darstellbar
    Das ist als würde man eine Übersichtskarte von Brasilien nehmen und sagen cool die bauen 20km breite Straßen.

  92. #92 Georg Hoffmann
    März 13, 2013

    @markus

    Da sich Onkel Georg nicht zu den Linke geäussert hat, nehm ick mal an, das diese Temperaturrekonstruktion soweit in Ordnung ist.”

    Das sind natuerlich Antarktis Rekonstruktionen. Global ist die Amplitude etwas kleiner.
    Jeder der Spruenge zum ENde einer Eszeit dauert 10000-15000 Jahre.

  93. #93 markus
    März 13, 2013

    @Georg
    Ja das verschärft die ganze Sache, letztlich ist es nichts Anderes als das Hereinzoomen bei Apfel und Birnen.
    Aber solche Graphen dienen dazu Informationen begreifbar zu machen.
    Ohne ein gewisses Maß an Vereinfachung geht es meistens nicht es sei den man hätte gern eine 60ge Jahre Tapete wie in. https://cdn1.spiegel.de/images/image-470000-galleryV9-lybo.jpg

  94. #94 Dr. Webbaer
    März 16, 2013

    @Hoffmann
    Wie passt es zusammen, wenn Milankovic-Zyklen u.a. als ursächlich für die Eiszeiten angenommen werden, dass atmosphärisches CO2 und terrestrische Oberflächentemperatur weitgehend zusammenlaufen oder mit hohem Faktor korrelieren?

    MFG
    Dr. W

  95. #95 Georg Hoffmann
    März 16, 2013

    @WB
    Nun, das scheint mir offensichtlich. Milankovitch macht etwas mit den globalen Temperaturen (oder denen eines Teils, etwa des suedlichen Ozeans) und das CO2/CH4 verstaerkt das (sofort, nach dem hier vorgestellten Paper oder mit einem Verzug) und so aendern sich die Temperaturen etc bis nach 10000 Jahren ein neues Gleichgewicht ohne Eisschilde erreicht ist.

  96. #96 Dr. Webbaer
    März 16, 2013

    @Hoffmann
    Sie merken, dass sich Ihr Kommentatorenfreund, der Webbaer, sozusagen für das Ruhesystem interessiert, das System ohne bedeutsamen anthropogenen Einfluss.
    Dieses hat gewisse Eigenschaften oder “Bandbreiten” die Entwicklung der terrestrischen Oberflächentemperaturen betreffend, legt aber explizit nicht nahe, dass wenn bestimmte, soz. besondere, Einflüsse das terrestrische Klima beeinflussen, stark beeinflussen, wir haben hier Schwankungen innerhalb eines 10K-Rahmens, der atmosphärische CO2-Gehalt zeitnah sozusagen mitspringt.

    Es wäre also für die historische Datenlage eine zumindest zeitweise Entkoppelung von atmosphärischem CO2-Gehalt und Oberflächentemperatur zu erwarten, die auch nachweisbar wäre, oder?

    MFG
    Dr. W

  97. #97 Georg Hoffmann
    März 16, 2013

    @DrWB
    Ich glaube nicht, dass das ein Satz ist (oder zwei). So oder so, ich verstehe es nicht.

    “Es wäre also für die historische Datenlage eine zumindest zeitweise Entkoppelung von atmosphärischem CO2-Gehalt und Oberflächentemperatur zu erwarten, die auch nachweisbar wäre, oder?”
    Verstehe die Praemisse nicht, da ich nicht verstanden habe, was Sie vorher sagten. Die Phasenlage zwischen CO2 und Temperatur koennen Sie im Paper nachlesen und ist hier zitiert. Fuer schnelle Schwankungen kommt man auf eine Unsicherheit von ca 100 Jahre. Wenn es aber nur langsam ungemaechlich wackelt bei der Temperatur und also auch beim co2(wie sehr lange Zeit im Glazial), dann wird natuerlich die Phasenlage unsicherer.

  98. #98 Dr. Webbaer
    März 16, 2013

    @Hoffmann
    Billiger formuliert: Wenn eine bestimmte Ursache wie bspw. ein von Milankovic erkannter Prozess für die Abkühlung der Erde in größerem Umfang verantwortlich ist, warum verändert dann der CO2-Gehalt der Atmosphäre zeitnah?

    MFG
    Dr. W

  99. #99 Georg Hoffmann
    März 16, 2013

    “Wenn eine bestimmte Ursache wie bspw. ein von Milankovic erkannter Prozess für die Abkühlung der Erde in größerem Umfang verantwortlich ist, warum verändert dann der CO2-Gehalt der Atmosphäre zeitnah?”
    Ein sich aendernder CO@ Gehalt aendert immer und erdgeschichtlich instantan die Energiebilanz der Erde. In den meisten Faellen nimmt das auch sehr schnell Einfluss auf die Temperatur. Es mag andere Faktoren geben, die gleichzeitig wirken und mit anderem Vorzeichen wirken, aber der EInfluss des CO2 auf die Energiebilanz ist, wie gesagt, instantan.

  100. #100 Dr. Webbaer
    März 16, 2013

    @Hoffmann
    Wenn ein sozusagen externer Prozess wie der Milankovic-Zyklus ursächlich im Sinne von großteilig oder hauptursächlich sein soll für die Bildung der Eiszeiten, Sie selbst könnten (nach einiger Web-Suche) dementsprechend verstanden werden oder worden sein, dann muss der atmosphärische CO2-Gehalt, der ja weitgehend mit den terrestrischen Oberflächentemperaturen (zeitnah) zu korrelieren scheint, hierzu eingeordnet werden.

    Der Schreiber dieser Zeilen sieht bisher nicht, dass das gelungen ist. [1]

    MFG
    Dr. W

    [1] wobei man dann auch anders folgern könnte, was die enge Korrelation von atmosphärischem CO2 und terrestrischer Oberflächentemperatur betrifft, nämlich symptomatisch; der Schreiber dieser Zeilen will das nicht tun, regt aber an Korrelationen bspw. im Sinne “A und B” standardisiert bspw. als “A wegen B” oder “B wegen A” oder “(A und B) wegen C” zu gruppieren

  101. #101 Georg Hoffmann
    März 16, 2013

    Wenn jemand mir helfen koennte. Ich verstehe nicht, worauf der WB, der sich sicher bemieht, hinaus will.
    Wenn der WB auf irgendetwas hinaus will wie: Woran sieht man, dass das CO2 wichtig war bei den EIszeiten, dann wuerde ich (neben anderen Dingen) hauptsaechlich auf die Eiszeitlichen Bedingungen in der Suedhemisphaere hinweisen. Dort gab es nicht so viel mehr Eis als heute (Geometrie), der Milankovitch Mechanismus wirkt ohnehin nur in der NH und ist auch fuer sie formuliert und der direkte Waermetransport ist nicht gross genug. Kurz, warum kuehlt die SH ab? Und da braucht man ganz klar (mal von der Groesze der Feedback etc abgesehen) die Treibhausgase.

  102. #102 Dr. Webbaer
    März 16, 2013

    @Hoffmann
    Wenn es historisch Schwankungen bei den terrestrischen Oberflächentemperaturen um die 10K gegeben hat, Stichwort: Eiszeiten, und diese offensichtlich nicht auf anthropogene Ursachen zurückgeführt werden können, dann entsteht vielleicht ein GANZ BESONDERER ANLASS die dennoch stattgefunden habende enge Korrelation (“mit hohem K-Faltor”) zwischen terrestrischer Oberflächentemperatur und atmosphärischem CO2-Gehalt zu erklären.

    MFG
    Dr. W (der es nicht weiter runterbrechen kann und sich aus anderen Gründen ohnehin ausklinken muss, dem hiesigen Webangebot natürlich weiterhin vorzüglichen Gedeih wünscht)

  103. #103 Georg Hoffmann
    März 16, 2013

    @WB
    Jetzt glaube ich, ich habe verstanden.
    “dann entsteht vielleicht ein GANZ BESONDERER ANLASS die dennoch stattgefunden habende enge Korrelation (“mit hohem K-Faltor”) zwischen terrestrischer Oberflächentemperatur und atmosphärischem CO2-Gehalt zu erklären.”
    Wir wissen dass die Temperatur alleine nicht die CO2 Schwankungen hinbekommt. Das ist ein relativ einfaches und unbezweifelte Konsequenz der Loeslichkeit von CO2 in Ozeanwasser. Es gibt aber riesig viele andere Mechanismen, die alle mit der Temperatur korrelieren (Meereisbedeckung, Staubeintrag in den Ozean, Ausdehnung der Schelfgebiete, Permafrostgebiete, und und und). Kurz mit den Temperaturen aendert sich der ganze Planet und jeder Faktor hat/kann haben einen Einfluss auf den Kohlenstoffzyklus und somit auf das CO2. Ich hoffe das ist jetzt die Antwort, die sie suchen.

    Im uebrigen war die letzte Eiszeit zwischen 4-6 Grad global kuehler (und sicher nicht 10 Grad) wobei die Tendenz in den letzten Jahren eher zu 4 als zu 6 Grad ging. Ersteres hat eher eine niedrige Klimasensitivitaet zur Folge, letzteres eine Grosse. 10 Grad kaelter waehrend der letzten EIszeit bedeutet ca eine Klimasensitivitaet von 6Grad pro CO2 Verdoppelung und waere natuerlich nicht so toll.

  104. #104 Dr. Webbaer
    März 17, 2013

    @Hoffmann
    Da kommt also eine Eiszeit, die nicht anthropogen zugeordnet werden kann, sondern stattdessen bspw. dem Milankovic-Zyklus, und es wird kälter, woraufhin der atmosphärische CO2-Gehalt zeitnah ebenfalls sinkt?

    So dass die festgestellte Korrelation zwischen atmosphärischem CO2-Gehalt und terrestrischer Oberflächentemperatur mit hohem Faktor gewahrt bleibt.

    Hmm, das kann so sein. Aber Sie erkennen vielleicht, dass gerade das vglw. strenge Zusammenlaufen von Temperatur und CO2 anti-intuitiv Argumente liefert eben nicht auf die bekannte Kausalität (“wenn CO2 hoch, dann allgemein warm”) zu schließen?!

    MFG
    Dr. W

  105. #105 Georg Hoffmann
    März 17, 2013

    @DrWB
    “Aber Sie erkennen vielleicht, dass gerade das vglw. strenge Zusammenlaufen von Temperatur und CO2 anti-intuitiv Argumente liefert eben nicht auf die bekannte Kausalität (“wenn CO2 hoch, dann allgemein warm”) zu schließen?!”

    Seien Sie mir nicht boese. Vielleicht beruht es ja auf Gegenseitigkeit und all die Probleme, die ich mit dem Verstaendnis ihrer Fragen habe, haben Sie mit meinen Antworten?
    Ich habe jetzt schon x-mal gesagt, dass im Rahmen eines zeitlichen Verlaufs (erst T und CO2, dann Meerespiegel, dann Staub etc etc) sich viele Dinge zum Ende einer EIszeit aendern, die sich alle Gegenseitig beeinflussen (Hoehere Ozean T, mehr CO2 ausgasen, hoehere T, mehr Eisschmelze und niedrigere Albedo, hoehere T etc etc). Es fuehlt sich fuer mich auesserst intuitiv richtig an wenn T und CO2 gleichzeitig schwanken, wobei gleichzeitig sowohl ±10 Jahr oder ±500 Jahre bedeuten kann.

  106. #106 Dr. Webbaer
    März 17, 2013

    @Hoffmann
    Also bspw. beim Beginn einer Eiszeit mit extraterrestrischer Ursächlichkeit ein erhöhter CO2-Verbrauch (neben anderen genannten Faktoren) durch die Pflanzenwelt und zum Ende einer Eiszeit, bei ebenfalls extraterrestrischen Ursächlichkeit, eine zeitnahe Zunahme des atmosphärischen CO2-Gehalts durch Ausgasung der Ozeane beispielsweise, korrekt?

    MFG
    Dr. W

  107. #107 axel
    März 17, 2013

    WUWT und andere Skeptikerblogs sind immer noch mit Marcott et al. vollauf beschäftigt, wenn die nur von Parrenin et al. wüssten, wäre der Infarkt nahe. Weiß nicht, ob ich mit dem Zählen hinterherkomme, ich bin bei der 8. “Widerlegung” bei WUWT angelangt.

    Warum? Skeptikerintuition. Da gibt’s ein Bild, das zeigt einen Hockeystick. Das muss falsch sein. Kurioserweise scheint man sich bei Marcott et al. an den letzten 100 Jahren festzubeißen und vergisst darüber vollauf, um was es eigentlich geht.

    PS:
    Früher hatte ich mal einen Bärchentransponder, damit ging es halbwegs.

  108. #108 axel
    März 17, 2013

    Die Diskussionsfreudigkeit in der NoBrainZone ging nach Georgs Beitrag sofort gegen null. Ganz schnell wegducken und vergessen, dass es Parrenin überhaupt gibt. Schade…

  109. #109 Georg Hoffmann
    März 17, 2013

    @axel
    Mir unerklæarlich. Wo doch der Gosselin sonst immer so eingruendlicher Forscher und Privatgelehrter ist, der den Dingen immer gnadenlos auf den Grund geht. Wie sonst waere er in den Beratungsstab der Kalten Sonne gekommen?

  110. #110 Georg Hoffmann
    März 17, 2013

    @WB
    Im Prinzip richtig, aber
    “Also bspw. beim Beginn einer Eiszeit mit extraterrestrischer Ursächlichkeit ein erhöhter CO2-Verbrauch (neben anderen genannten Faktoren) ”
    Das Beispiel ist nicht korrekt. Bei einer beginnenden Eiszeit schrumpft die aktive Biomasse und nicht umgekehrt. Aber wie gesagt, das Prinzip ist richtig.

  111. #112 Dr. Webbaer
    März 19, 2013

    @Axel
    Erst einmal liegt die Mischung von Datenlage und Prognose vor, sowas ist nicht so-o cool, und wenn die Prognose falsch ist oder werden wird, ist sie auch alarmistisch oder wird alarmistisch.

    Es ist einfach so, dass zwischen 1880/01 und 2013/01 eine durchschnittliche Erwärmung der terrestrischen Oberflächentemperaturen von ca. +0,06K / Dekade belegt ist und die zeitgenössische Prognostik eine Erwärmung von ca. +0,36K benötigt, um wahr zu sein oder wahr bleiben zu können.

    In jedem Wirtschaftsbetrieb würde man jeder Vertriebskraft in einer vergleichbaren Situation, bspw. das Auseinanderlaufen von nachgewiesenen und prognostizierten Umsatzzahlen betreffend, die Ohren se-ehr langziehen und womöglich auf die weitere Mitwirkung verzichten.

    MFG
    Dr. W

  112. #113 Georg Hoffmann
    März 19, 2013

    @WB
    “Es ist einfach so, dass zwischen 1880/01 und 2013/01 eine durchschnittliche Erwärmung der terrestrischen Oberflächentemperaturen von ca. +0,06K / Dekade belegt ist und die zeitgenössische Prognostik eine Erwärmung von ca. +0,36K benötigt, um wahr zu sein oder wahr bleiben zu können.”

    Es wird durch Wiederhohlen nicht wahr. IPCC Erwaermungsraten sind relativ zu 1980-1999. Modelle koennen den bekannten Erwaermungstrend des 20ten Jhd ganz gut nachbilden, machen aber dazu Annahmen ueber Dinge (Aerosol Konzentration und auch anderes) die in der Tat Unsicherheiten haben und nicht wie man es sich wuenschen wuerde bekannt sind.
    Zu den “noetigen” IPCC kompatiblen Erwaermunsgraten fuer die Zukunft siehe meine Antwort an Sie oben.

  113. #114 shader
    März 19, 2013

    Es ist einfach so, dass zwischen 1880/01 und 2013/01 eine durchschnittliche Erwärmung der terrestrischen Oberflächentemperaturen von ca. +0,06K / Dekade belegt ist und die zeitgenössische Prognostik eine Erwärmung von ca. +0,36K benötigt, um wahr zu sein oder wahr bleiben zu können.

    @Webbaer, diese Aussage haben Sie hier schon zu Genüge wiederholt. Aber was wollen Sie damit ausdrücken. Im übrigen ist kein Temperaturanstieg von 0,36K pro Jahrzehnt nötig, um Klimaprognosen zu bestätigen. Sie reduzieren Aussagen aus der Klimawissenschaft auf einen einzelnen Punkt. Das wäre so, als wenn man die klassische Physik auf die Aussage reduzieren würde, Äpfel fallen immer nach unten.

  114. #115 Dr. Webbaer
    März 19, 2013

    @Hoffmann
    Wahr ist die hier in zweiter Schicht zitierte Aussage schon:

    Es ist einfach so, dass zwischen 1880/01 und 2013/01 eine durchschnittliche Erwärmung der terrestrischen Oberflächentemperaturen von ca. +0,06K / Dekade belegt ist und die zeitgenössische Prognostik eine Erwärmung von ca. +0,36K benötigt, um wahr zu sein oder wahr bleiben zu können. [Dr. W]

    Es wird durch Wiederhohlen [sic] nicht wahr. IPCC Erwaermungsraten sind relativ zu 1980-1999. Modelle koennen den bekannten Erwaermungstrend des 20ten Jhd ganz gut nachbilden, machen aber dazu Annahmen ueber Dinge (Aerosol Konzentration und auch anderes) die in der Tat Unsicherheiten haben und nicht wie man es sich wuenschen wuerde bekannt sind. [Dr. H]

    Verstehen Sie bitte die getätigte Aussage nicht dahingehend, dass die aktuelle Prognosis falsch sein muss, sie bleibt aber im Verhältnis zum Gemessenen -wie (gut) begründet auch immer- weitgehend oder im Verhältnis von ca. “5:1” theoretisch.

    Man könnte sich sicherlich darauf einigen, dass die (wohlbegründete) Theorie der Praxis ein wenig voraushinkt, vielleicht um den Faktor 6. [1]

    MFG
    Dr. W

    [1] wobei bei Faktor 10 die sogenannte Webbaersche Vermutung greifen könnte, die die Rezipienz zur allgemeinen Ablehnung anleiten könnte – all dem könnte man durch eine rechtzeitige Absteifung bei der Prognosehöhe und einem Mehr vonstorchiger Vortragsweise bestmöglich vorbeugen *

    * Dr. W durchaus in dem Sinne voll auf Ihrer Seite, dass das klimatologistische Bemühen grundsätzlich angemessen ist

  115. #116 Georg Hoffmann
    März 19, 2013

    @WB
    “Verstehen Sie bitte die getätigte Aussage nicht dahingehend, dass die aktuelle Prognosis falsch sein muss, sie bleibt aber im Verhältnis zum Gemessenen -wie (gut) begründet auch immer- weitgehend oder im Verhältnis von ca. “5:1″ theoretisch.”

    Nochmal: Nein. Die jetzigen Vorraussagen fuer das 21 Jhd koennen nicht mit den Beobachtungen des 20ten verifiziert werden, sondern mit den Simulationen des 20ten Jhd. Ich kanns nicht aendern. Dass Sie etwas mit etwas belegen oder falsifizieren moechten, bedeutet nicht, dass das sinnvoll ist.

  116. #117 Dr. Webbaer
    März 19, 2013

    @Hoffmann

    Die jetzigen Vorraussagen fuer das 21 Jhd koennen nicht mit den Beobachtungen des 20ten verifiziert werden

    Es geht selbstverständlich weder um die Veri- wie Falsifizierung, sondern um den nackten Vergleich von bisher Gemessenem und bisher Theoretisiertem.

    Dabei ist die Datenlage dünn, man misst anscheinend beim NASA Goddard Institute for Space Studies (GISS) seit 1880/01, die ersten Messreihen sind mit Vorsicht zu genießen, vielleicht sogar bis in die 30er-Jahre hinein, aber es gibt hier nun einmal dieses Auseinanderlaufen.

    Wie man das nun genau spezifiziert, manche mögen sich am ‘IPCC Erwaermungsraten sind relativ zu 1980-1999.’ (Bonusfrage: Warum genau, weil’s gefällt?) festhalten, …, …, … nun, es konveniert nicht in dem Sinne, dass Theorerisierung und Messung nicht auseinanderlaufen.

    Ist doch auch nicht schlimm, und was macht man da idR?, man passt die Prädiktion ein wenig an, gibt sich vielleicht in Vonstorchscher Hinsicht autokritisch und nach Absteifung der Prognosis hat man dann mehr Zeit gegenzuwirken.

    Gerne auch per Geo-Engineerung, auch wenn R. das gar nicht mag.

    MFG
    Dr. W

  117. #118 Georg Hoffmann
    März 19, 2013

    @WB
    “sondern um den nackten Vergleich von bisher Gemessenem und bisher Theoretisiertem.”
    Ja, das nennt man Verifikation.
    Und nochmals: Es gibt KEINE THEORIE oder Aussage, dass die Temperaturgradienten des 20ten Jhd 0.34 Grad pro Dekade sein muessten. Sie haengen natuerlich vom Gesamtforcing ab und das war ueber weite Zeit nicht vom CO2 dominiert.

    “Wie man das nun genau spezifiziert, manche mögen sich am ‘IPCC Erwaermungsraten sind relativ zu 1980-1999.’ (Bonusfrage: Warum genau, weil’s gefällt?) festhalten,”
    Weil die Modelle so aufgesetzt wurden. Wo glauben sie denn, dass diese Trends die Sie fuer verschiedenen Scenario genannt haben, herkommen? Haben Sie das entsprechende Kapitel gelesen, dass erklaert wie diese Trends berechnet wurden? Oder nehmen Sie irgendwelche Zahlen und verleichen Sie (weil es Ihnen gerade so gefaellt) mit den Temperaturschwankungen des fruehen 20ten Jhd?

    Reach you potential. Auch Sie koennen sicher ein Argument sauber recherchieren und dann formulieren.

  118. #119 Dr. Webbaer
    März 19, 2013

    @Hoffmann

    Reach you potential. Auch Sie koennen sicher ein Argument sauber recherchieren und dann formulieren.

    Haha, das tut doch Ihr Kommentatorenfreund seit Längerem. Er nimmt Sie doch ernst nachdem Sie sich vom Ökologismus-Verdachts weitgehend entledigen konnten.

    Es gibt KEINE THEORIE oder Aussage, dass die Temperaturgradienten des 20ten Jhd 0.34 Grad pro Dekade sein muessten.

    Man benötigt nun jedenfalls, für das 21. Jahrhundert eine Zunahme bei der terrestrischen Oberflächentemperatur von ca. +0,36K / Dekade um den Vorhersagerahmen von ca. +2K bis +6K bis 2100, siehe IPCC, siehe Mudelsee, siehe alles was rumspringt, erreichen zu können.

    MFG
    Dr. W (der sich aber nun, auch wegen einer gewissen Regsamkeit, gerne bis auf Weiteres ausklinkt, auch nicht stören wollte)

  119. #120 Georg Hoffmann
    März 19, 2013

    @WB
    “Man benötigt nun jedenfalls, für das 21. Jahrhundert eine Zunahme bei der terrestrischen Oberflächentemperatur von ca. +0,36K / Dekade um den Vorhersagerahmen von ca. +2K bis +6K bis 2100, siehe IPCC, siehe Mudelsee, siehe alles was rumspringt, erreichen zu können.”

    Ich komme mir langsam vor, wie in der stationaeren Psychiatrie.
    Die Vorhersage ist also von 1980-1999 im Verglecih zu 2090-2100. Sie kann nicht eingeordnet/verglichen/falsifiziert/sinnvoll gegenuebergestellt werden zum gesamten 20 Jhd. Denn dann muesste man fragen: Warum denn das 20 Jhd und nicht das 16te? Das Gesamtforcing des 20ten Jhd ergibt durch die gleichen Modelle, die Ihnen die Erwaermung des 21ten Jhd berechnen (also ihre 2-6 Grad), dargestellt einen recht sinnvollen Temperaturverlauf.
    Dazu mehr:https://scienceblogs.de/primaklima/2010/09/04/model-ranking-kann-man-aus-den-fehlern-von-gestern-lernen/

  120. #121 axel
    März 19, 2013

    @ Bärchen

    In der in #111 verlinkten Graphik ist eindeutig in der Legende beschrieben, was Daten und was Prognose ist. Wir sollten den Betrachtern durchaus ein Minimum an Lesekompetenz zutrauen, meine ich.

    Ich finde die Graphik gelungen, weil sie den gegenwärtigen Klimawandel mal auf einer geologischen Zeitskala zeigt. Der Übergang von Glazial zum Holozän war da ein richtiger Klacks dagegen, oder?

    Und: Man kann ja einfach mal von einem sehr, sehr günstigen Fall ausgehen, und zwar von einem sehr niedrigen Wert der Klimasensitivität von 2°C. Ändert sich etwas grundlegend? Ich meine nicht.

  121. #122 shader
    März 19, 2013

    “Man benötigt nun jedenfalls, für das 21. Jahrhundert eine Zunahme bei der terrestrischen Oberflächentemperatur von ca. +0,36K / Dekade um den Vorhersagerahmen von ca. +2K bis +6K bis 2100, siehe IPCC, siehe Mudelsee, siehe alles was rumspringt, erreichen zu können.”

    Mal ganz langsam zum Mitmeißeln für Nichtbären, warum muss ein dauerhafter Temperaturanstieg von 0,36K/Dekade erzielt werden, um eine Aussage, dass bis 2100 bei einem bestimmten Szenario die Temperatur um minimum 2K ansteigt, bestätigt wird?

  122. #123 Dr. Webbaer
    März 19, 2013

    Mal ganz langsam zum Mitmeißeln für Nichtbären, warum muss ein dauerhafter Temperaturanstieg von 0,36K/Dekade erzielt werden, um eine Aussage, dass bis 2100 bei einem bestimmten Szenario die Temperatur um minimum 2K ansteigt, bestätigt wird?

    Weil die durchschnittliche Prognosis ca. +3,2K bis 2100 (bzw. die jeweils letzten Jahrzehnte des 20. und des 21. Jhd. betreffend) erwartet und nur noch 86-87 Jahre bereitstehen und so eine Erwärmung von ca. 0,36K / Dekade benötigt wird, um der Prognoseleistung zu entsprechen?

    MFG
    Dr. W (der allerdings ob der erweiterten Erklärungen zur Webbaerschen Vermutung, und um die geht es hier hintergründig, langsam etwas müde wird)

  123. #124 axel
    März 19, 2013

    @ S. Hader

    Bärchen kann nur in linearen Funktionen denken und “argumentieren”. Haben wir ihm schon 100x erklärt, 100x ist nichts passiert. Glauben Sie wirklich, mit dem 101. Versuch mehr Erfolg zu haben? 😉

  124. #125 Dr. Webbaer
    März 19, 2013

    @Axel
    Gehen Sie mal davon aus, dass die Abnehmerschaft, also die Politik und indirekt auch das Wahlvieh, anfangen wird zu murren, je klarer die Entwicklung bei den Oberflächentemperaturen von den nun benötigten ca. +0,36K / Dekade nach unten abweicht.

    Die Theoretisierung mag ja OK sein, der Schreiber dieser Zeilen glaubt das nicht umfänglich, aber das tut wenig zur Sache, wenn die Datenlage sozusagen die Theorie frisst.

    MFG
    Dr. W

  125. #126 shader
    März 19, 2013

    @Webbaer: Warum muss haargenau eine durchschnittliche Temperatursteigung stattfinden, wenn Sie selbst das Intervall von 2 bis 6 K angeben? Eine Bestätigung der These würde auch dann stattfinden, wenn bis 2100 (bei Einhaltung der Randbedingungen) die Durchschnittstemperatur tatsächlich >= 2K gestiegen ist. Zudem ist es auch sehr wahrscheinlich, dass von Jahrzehnt zu Jahrzehnt die Anstiege erheblich schwanken können. Simple Logik. Verstehen auch promovierter Forenschreiber. ^^

  126. #127 axel
    März 19, 2013

    @ Bärchen

    Nun ist es aber so, dass die breite Öffentlichkeit von den Webbärchen-0,36°C/Dek. gar nichts weiß. Und selbst wenn sie es wüsste, man würde wohl eher auf IPCC-Angaben hören (ich schätze, Sie haben die Stelle in der SPM des AR4 inzwischen gefunden).

    PS:
    Uns ist Bärchens Grundschulmathematik auch egal.

  127. #128 Dr. Webbaer
    März 20, 2013

    @Shader

    Eine Bestätigung der These würde auch dann stattfinden, wenn bis 2100 (bei Einhaltung der Randbedingungen) die Durchschnittstemperatur tatsächlich >= 2K gestiegen ist. Zudem ist es auch sehr wahrscheinlich, dass von Jahrzehnt zu Jahrzehnt die Anstiege erheblich schwanken können.

    Jaja, und darum wurde ja auch der Durchschnittswert angegeben. Der liegt bei ca. +0,36K / Dekade. Sie haben es langsam geschnallt. Chapeau.

    Aber man könnte hier noch verhandeln, der Spielraum wäre ca. +0,2K / Dekade bis +0,6K / Dekade, wobei es +0,36K / Dekade des Bären Erachtens aber gut trifft.

    Wobei zwischen 1880/01 und 2013/01 aber nur eine durchschnittliche Erwärmung der terrestrischen Oberflächentemperaturen von +0,06K / Dekade stattgefunden hat. [1]

    MFG
    Dr. W

    [1] Achtung: Das ist eine primär politische Aussage *, wenn sie hier vergleichend angelegt wird. – Vielleicht nagt es den einen oder anderen diesbezüglich im Verständnis.

    * die Rezipienz betreffend, die Grillbedarf entwickeln könnte
    >:->

  128. #129 Leugnerentlarver
    März 20, 2013

    Also ich finde die Prophezeiungen von Dr. Webbaer, auf der Basis der Voraussagen im PIK, absolut korrekt gerechnet.

    Man hat im PIK aber auch schon von höheren T Anstiegen spekuliert, sodass auch 0.50 C / Dekade Anstieg möglich erscheinen.

    Das wird ja sehr spannend. Hoffentlich schlägt kein Meteorit ein.

  129. #130 Dr. Webbaer
    März 20, 2013

    @Leugnerentlarver
    Ja, die Jungs hier stellen sich einerseits dumm, andererseits unterstellen sie, dass ihr Kommentatorenfreund, der Webbaer, klimatologistisch ausgesagt hätte, statt politisch – was die bisherigen Durchschnittswerte und die übliche Prognosis betrifft.

    Das kann ja auch jeder nachrechnen, es ist exoterisch.

    MFG
    Dr. W

  130. #131 Georg Hoffmann
    März 20, 2013

    @WB,Leugnerentlarver
    Ich bin absolut damit einverstanden, dass eine vorhergesagte Erwaermung X bis zum Ende des Jhd sich in einem Erwaermungstrend von Y±Z widerspiegeln muss. Aber warum nicht die Zahlen korrekt ausrechnen? Ich habe es oben vorgemacht.
    1) Erwaermung ist relativ zu 1980-1999. Also von allem 0.3 abziehen.
    2) Nicht ueber verschiedene Scenarios mitteln. Das ist sinnlos. Es gibt kein Mittel zwischen einer Welt mit 9 und einer mirt12 Milliarden Menschen in diesem Zusammenhang.

    Also nehmen wir mal das Szenarion A1B. Das macht 2.1-3.4 Grad. Minus 0.3. Also 0.18C-0.31C pro Dekade ± dekadischer Variabilitaet, also ca 0.15 oder so (alle Zahlen aus der Erinnerung).

  131. #132 Dr. Webbaer
    März 20, 2013

    @H-Man

    Also 0.18C-0.31C pro Dekade ± dekadischer Variabilitaet, also ca 0.15 oder so (alle Zahlen aus der Erinnerung).

    Das scheint etwas niedrig angesetzt. Es gibt -aus politischer Sicht- eine Szenarienmenge, die -wiederum: politisch- gemittelt werden darf, und die 0,15K / Dekade sind hier zu wenig, um der Erwartung des Wahlviehs zu entsprechen.

    Man geht schon allgemein von 2K bis 6K bis 2100 [1] aus, die genannten Vergleichszeiträume berücksichtigend, und davon aus, dass man nun noch 87 oder 77 Jahre hat vergleichsweise zur Prognosis zu messen.

    MFG
    Dr. W

    [1] Erinnernd: Man will bei CO2-Reduktion und allgemeiner “Großer Transformation” auf +2K bis 2100 kommen und dann die Entwicklung in der Folge vielleicht bei +3K stoppen.

  132. #133 Georg Hoffmann
    März 20, 2013

    @WB
    “Das scheint etwas niedrig angesetzt. Es gibt -aus politischer Sicht- eine Szenarienmenge, die -wiederum: politisch- gemittelt werden darf, und die 0,15K / Dekade sind hier zu wenig, um der Erwartung des Wahlviehs zu entsprechen.”

    Gut, da faellt mir nicht mehr viel zu ein. Mitteln Sie also weiter politisch und ich mittele arithmetisch ueber Dinge die a priori gleich wahrscheinlich sind, wenn das ok ist.
    Thou shall not average over different scenarios!

  133. #134 Georg Hoffmann
    März 20, 2013

    Und mit den 0.15 Grad habe ich die dekadische Variabilitaet des Trends angegeben und nicht den Trend selbst. Thou shall read what you comment!

  134. #135 Dr. Webbaer
    März 20, 2013

    @H-Man
    Alles gelesen, die Variabilität ist erkennbar höher als 0,15K – und 0,18K bis 0,31K sind auch etwas zu wenig.

    BTW, wann genau werden Sie englisch-sprechend?

    MFG
    Dr. W (der sich aber nun, auch wegen einer gewissen Regsamkeit, gerne bis auf Weiteres ausklinkt, auch nicht stören wollte)

  135. #136 axel
    März 20, 2013

    @ Leugner.

    Also ich finde die Prophezeiungen von Dr. Webbaer, auf der Basis der Voraussagen im PIK, absolut korrekt gerechnet.

    Ist mir ziemlich egal. Dann gibt’s halt zwei Dumme hier.

    PS:
    Das PIK soll +0,5 für diese und die nächste Dekade für möglich halten? Bla bla, da verwechseln Sie etwas. Wie wär’s mit ‘ner Quellenangabe?

  136. #137 Leugnerentlarver
    März 20, 2013

    Axel, meinte Sie etwa 2 Dumme und 6 Vollidioten?

  137. #138 ijoe
    März 22, 2013

    Die Datierungsmethoden C14, Argon, Dendro, Eiskern, sind allesamt nicht zu gebrauchen, da zirkulär. Siehe auch “C14-Crash” und “Ceno-Crash”.

  138. #139 Adent
    März 22, 2013

    Ja iJoe, das haben schon viele Esos vor dir rumposaunt, wie wäre es mal mit Beweisen für diese absurde Behauptung?

  139. #140 rolak
    März 22, 2013

    da zirkulär
    ..
    wie wäre es mal mit Beweisen

    Hast Du doch selber elegant erledigt, Adent: Das ist wahr, weil es schon viele vor ihm ausposaunt haben.

  140. #141 ijoe
    März 22, 2013

    Wie gesagt, die Beweise kann man nachlesen in den Büchern “C14-Crash” und “Ceno-Crash”.

  141. #142 Günther Vennecke
    März 22, 2013

    @ijoe

    Können Sie nicht bis zum 1. April warten, ehe Sie mit einem solchen Zeugs kommen?

    “Der in Berlin lebende Diplom-Physiker Christian Blöss (*1957 in Kiel) ist seit Beginn der 1980er Jahre ein überzeugter Verfechter wissenschaftlicher Kritik an der Darwin-Haeckelschen Evolutionstheorie und gehört inzwischen zu den führenden Forscher-Persönlichkeiten des Neo-Katastrophismus im deutschsprachigen Raum. Diesbezüglich geht er von kataklysmischen Ereignissen als Folge von Nahbegegnungen der Erde mit anderen planetaren Körpern aus.”

    Das ist doch ein esoterischer Spinner, mit dem man sich nun wirklich nicht mehr inhaltlich auseinandersetzen muss. Auslachen reicht völlig.

  142. #143 Günther Vennecke
    März 22, 2013

    Den Ceno-Crash gibt es als pdf online:

    https://www.paf.li/CenoCrash.pdf

    Dieser eine Satz aus dem Vorwort verrät schon den absoluten Spinner, möglicherweise ein Kreationist:

    “Die Koexistenz von Menschen und Dinosauriern wird greifbar.”

    Ein weiteres Beschäftigen mit den Inhalten des Buches wäre reine Zeitverschwendung, trotzdem bleibt es natürlich jedem unbenommen, genau dies zu tun.

  143. #144 Günther Vennecke
    März 22, 2013

    Auch den “C14-Crash” gibt es als pdf:

    https://91.210.216.1/Books/%D0%9D%D0%A5/c14crash.pdf

    Auch hier zeigt ein kurzes Anlesen, wie spinnert und ahnungslos der Autor ist:

    In den neunziger Jahren entdeckte Heribert Illig, von Angelika Müller und einem der Autoren (HUN) auf Verwunderungen und Rätsel der Mittelalterhistoriker hingewiesen, daß auch das Mittelalter seine Phantomzeit hat. Und zwar müssen mindestens die drei Jahrhunderte zwischen ca. 600 und 900 u.Z. als frei erfunden gelten.

    Herbert Illig muss ebenso zu den Spinnern gerechnet werden, da er tatsächlich ein ganzes Buch über angebliche Phantomjahrhunderte geschrieben hat, wo doch völlig klar ist, dass unsere Geschichtsschreibung hier keine Fehler aufweist.

    Das lässt sich für einen an Klimaforschung interessierten leicht belegen:

    In Eisbohrkernen, deren Schichten man zumindest für etliche Jahrhunderte der jüngeren Vergangenheit oft auf ein Jahr genau datieren kann, finden sich auch Ascheablagerungen von größeren Vulkanausbrüchen, deren Ausbruchsjahr historisch bekannt ist. So ist es kein Problem, fest zu stellen, ob in den Eisschichten, die aus dem Jahr 79 u. Z. und kurz danach datieren, Ascherückstände vom Ausbruch des Vesuvs zu finden sind, der seinerzeit zur Zerstörung von Pompeji und Herculaneum geführt hat.

    Wenn diese Schichte also 1934 Jahre alt sind ist damit ist glasklar belegt, dass unsere Zeitrechnung NICHT durch Illigs beknackte “Phantomjahrhunderte” angereichert wurden.

    Es ist wirklich erschreckend, wie viele Zeitgenossen immer wieder auf solche Spinner wie Illig, Blöss, u. a. hereinfallen.

  144. #145 ijoe
    März 24, 2013

    Zu den Eisbohrkernen: Heinsohn, Gunnar (1994) : “Für wieviele Jahre reicht das Grönlandeis?”, in Zeitensprünge (vormals Vorzeit-Frühzeit-Gegenwart).
    Pompeji ist ein gutes Beispiel: aufgrund der “gesicherten” Datierung paßt natürlich auch der Eisbohrkern mit seinen “Jahresringen”, die natürlich nichts anderes als Schneefälle sind.

  145. #146 Günther Vennecke
    März 24, 2013

    @ijoe,

    Sie haben sich da eine kleine Gruppe von selbst ernannten “Experten” ausgesucht, die allesamt nicht wissen, wovon sie eigentlich reden.

    Diese Typen (Blöss, Illig, Heinsohn) haben nicht den Schimmer einer Ahnung von Datierungsmethoden und behaupten einen Unsinn nach dem anderen. Typisch Geistes-“Wissenschaftler”, die mit den Erkenntnissen und Methoden der Naturwissenschaft gewaltig auf Kriegsfuß stehen.

  146. #147 Günther Vennecke
    März 24, 2013

    @ijoe,

    es ist schon im höchsten Maße lächerlich, wenn ein paar dahergelaufene Möchtegern-Experten meinen, sie könnten vielfach getestete Datierungsmethoden mit äußerst laienhaften “Bedenken” ad absurdum führen.

    Dabei geben sie eigentlich nur zu erkennen, dass sie von wissenschaftlichen Methoden nicht den Schimmer einer Ahnung haben. Sie “übersehen” dabei geflissentlich, dass es i. d. R. mehrere, voneinander völlig unabhängige Wege gibt, um das Alter eines Artefaktes oder einer Sedimentschicht zu ermitteln.

    Alles, was die “Kritiker” gegen C14, Dendrochronologie, u. v. a. vorbringen, ist unausgegorener Schwachsinn, der die wirklichen Kenner der Methoden bestenfalls zu Lachkrämpfen animiert, ob der Dummheit oder Dreistigkeit, mit der die “Bedenken” vorgetragen werden.

    Wenn Ihre “Experten” irgendetwas von Belang vorzutragen hätten, könnten sie sich einem peer-review-Prozess unterwerfen. Sie wissen aber ganz genau, dass sich nicht einmal beim Pförtner des jeweiligen Verlages ernst genommen würden, weshalb sie sich der wissenschaftlichen Diskussion gar nicht erst stellen, sondern lieber in den privat-wirtschaftlich geführten Medien und bei zweifelhaften Verlagen (die allen Mist veröffentlichen, wenn er nur Gewinn verspricht) Wissenschaftsschelte à la Däniken betreiben.

    Solche Spinner hat es schon immer gegeben und wird es wohl auch immer geben (siehe Klimaleugner), aber sie verdienen nichts besseres, als dass man sie links (äh, rechts) liegen lässt und zur Tagesordnung übergeht.

    Ein Blöss, ein Heinsohn und ein Illig lassen sich problemlos schon von interessierten Laien widerlegen, was beweist, dass man schon ziemlich uninformiert und kritiklos (also das genaue Gegenteil von einem Skeptiker) sein muss, um diesen Schwachköpfen, bzw. skrupellosen Geschäftemachern* auf den Leim zu gehen.

    *) Es gibt tatsächlich nur zwei Möglichkeiten:

    1. diese Typen glauben den Mist, den sie erzählen, wirklich selbst, dann sind sie unglaublich dumm

    2. sie wissen ganz genau, dass sie Mist erzählen, dann sind sie skrupellose Geschäftemacher, die die Dummheit gewisser Mitmenschen schamlos ausnutzen, um sich zu bereichern.

    Persönlich tendiere ich zu 2.

    Tertium non datur.

  147. #148 ijoe
    März 27, 2013

    “Alles, was die “Kritiker” gegen C14, Dendrochronologie, u. v. a. vorbringen, ist unausgegorener Schwachsinn, der die wirklichen Kenner der Methoden bestenfalls zu Lachkrämpfen animiert, ob der Dummheit oder Dreistigkeit, mit der die “Bedenken” vorgetragen werden.”

    Was genau ist Schwachsinn, kannst Du da was zu sagen?

  148. #149 Leugnerentlarver
    März 29, 2013

    Herr Vennecke, kann es sein, dass Sie übermässig autoritätsgläubig sind? Sie glauben ja offenbar alles, was man Ihnen vorsetzt, bzw. was Ihrer ideologischen Vorprägung als Grüner am besten in den Kram passt.

    Und alle, die nicht Ihrer ideologisiert vorgefassten Meinung sind, bezeichnen Sie als Lügner, Gauner und Ganoven.

  149. #150 Klimarealist
    März 29, 2013

    @Leugnerentlarver
    Wegen Illig habe ich mich in den 80iegern noch im Fido-Net mit einem gewissen Lelarge schon demaßen was von in den Haaren gehabt, die haben echt einen Knall.
    Der Illig glaubt den Quatsch wirklich, aus seiner Sicht hört sich das alles sehr “vernünftig” an (habe den Fehler begangen, mir sein Buch zu kaufen, um es widerlegen zu können ), tatsächlich ist das alles der reinste Krampf.

    Und alle, die nicht Ihrer ideologisiert vorgefassten Meinung sind, bezeichnen Sie als Lügner, Gauner und Ganoven.

    Uneingeschränkte Zustimmung !!

    C14 hat natürlich seine Schwächen, jede Menge sogar, wenn man die aber kennt und einbezieht, dann ist das wiederum ok.

  150. #151 Klimarealist
    März 29, 2013

    @Leugnerentlarver

    Uneingeschränkte Zustimmung !!

    Natürlich nicht zu Venneckes Aussage !!!!!
    Sondern zur Kritik an diesem Individuum.

  151. #152 ijoe
    Juni 18, 2013

    “C14 hat natürlich seine Schwächen, jede Menge sogar, wenn man die aber kennt und einbezieht, dann ist das wiederum ok”.
    Eben nicht, es sind grundsätzliche Fehler, so daß alles, was daraus folgt, falsch ist.
    BTW gabs eigentlich schon Korrekturen der Historiker nach dem ProtschvonZieten-Skandal?

  152. #153 festkl?nningarp
    https://www.iamsport.org/pg/blog/fightcereal8/read/16251308/much-more-2013-new-wi-dells-points-of-interest-presented
    Juli 12, 2013

    Great Article! I’ve been searching for this tonight and sooo want to get an email or the updates for this article. Stay in touch!

  153. #154 weltklima - Seite 240
    September 18, 2013

    […] einer Vereisung liegen die Linien aufeinander. Der angebliche zeitliche Abstand von 800 Jahren ist alles andere als sicher. […]

  154. #155 Karl Schmitt
    August 11, 2017

    Wir kommen gerade aus der kleinen Eiszeit und man wundert sich warum es wärmer wurde???Nun wenn es nicht wärmer geworden wäre wären wir noch immer in der kleinen Eiszeit.

    Wenn es den Ökos um Co2 geht,warum wurden dann die Billionen die uns die Co2 Vermeidung bsiher gekostet hat nicht in dne Kauf von Regenwäldern/Wäldern oder Aufforstung gesteckt?

  155. #156 Horst
    Brunsbuettel
    August 15, 2017

    Hallo,
    ich hätte da zwei fragen, ich hoffe man kann sie mir beantworten:
    1. Es wurde der Nachlauf der CO2-Konz. thematisiert und dies mit atmosphärischem Austausch im Firneis erklärt.
    Dabei verstehe ich jedoch nciht, wieso es dann nachläuft. Das CO2 müsste dann vorlaufen, weil: Angenommen Temperatur und CO2 steigen zeitlich, dann würde die höhere CO2-Konzentration in das noch nicht fest sedimentierte Firneis eindringen und damit historische Eisschichten mit CO2 anreichern. Dadurch hat die momentane CO2-Konz. einfluss auf das im Eis gebundene CO2, die CO2-Konz im Eis muss also am Ende eine Kurve ergeben, die bei zeitgleicher tatsächlicher Erhöhung ein Vorlaufen des CO2-Anstiegs abbildet.

    2. In historischen Eiskernen sieht man nur sehr wenige “Zacken”, sie sind offensichtlich durch Diffusion geglättet. Demnach dürfte man doch niemals den CO2-Peak von heute (falls es nur ein Peak ist) mit historischen Eiskernen vergleichen.

    Danke fuer die Aufklaerung!
    Horst

  156. #157 De Maigus
    Dezember 16, 2019

    Tja, die Eusbohrkerndaten mal wieder, die einzige Datenquelle, die überhaupt eine Korrelation zwischen CO2 und Temperatur zeigt (alle anderen Datenquellen zeigen ganz klar, dass es überhaupt keine Korrelation gibt). Und diese Daten muss man sich also so lange schönrechnen, bis das rauskommt, was man zeigen will… Die real gemessenen Werte auf der Erde zeigen klar, dass es keine Korrelation gibt. Falls Sie mich eines Besseren belehren wollen, nennen Sie mir bitte den Korrelationskoeffizienten inkl. Rechenweg (keine Angst, als Diplom Wirtschaftsinformatiker verstehe ich, wie man den berechnet). Wenn Sie schon dabei sind, können Sie mir dann gleich noch erklären, wie es zur mittelalterlichen Warmzeit kam (CO2 niedriger als heute – Temperatur deutlich wärmer als heute, Grönland war komplett grün).

  157. #158 Gmendel
    Dezember 19, 2019

    Grönland im Mittelalter komplett grün? Werner bist du’s?